NOTEPAD
Results
of 200 questions answered correctly

You have reached of 200 points, ( %)

Your time

Question 1 of 200

1. The infectious diseases department of a hospital admitted a patient with nausea, liquid stool with mucus and blood streaks, fever, weakness. Dysentery was suspected. What method of laboratory diagnostics should be applied to confirm the diagnosis?

Explanation

The laboratory diagnosis of infectious diseases involves two main approaches: one is the bacteriologic approach in which the organism is identified by staining and culturing the organism and the other is the immunologic approach in which the organism is identified by detection of antibodies against the organism in the patient’s serum. When cultures are negative (i.e bacteriologic method), then immunologic testing is commonly used. 

2. Malaria is treated with structural analogs of vitamin B2 (riboflavin). These drugs disrupt the synthesis of the following enzymes in plasmodium:

Explanation

Vitamin B2 (riboflavin): component of flavoproteins FAD and FMN, used as a cofactor in redox reactions (both aerobic and anaerobic dehydrogenases and oxidases).

Flavine mononucleotide – FMN

Flavine adenine dinucleotide – FAD

[NAD – Vitamin B3 (niacin, PP- pellagra preventing)].

3. A 6-year-old child with suspected active tuberculous process underwent the diagnostic Mantoux test. What immunobiological preparation was injected?

Explanation

Tuberculin (Mantoux) skin test: this test is done by intradermal injection of tuberculoprotein (tuberculin), purified protein derivative (PPD). Type IV hypersensitivity reaction.

 

Immunization against tuberculosis is induced by injection of attenuated strains of bovine type of tubercle bacilli, Bacilli Calmette Guerin (BCG). 

4. In a young man during exercise, the minute oxygen uptake and carbon dioxide emission equalled to 1000 ml. What substrates are oxidized in the cells of his body?

Explanation

Minute O2 uptake – 1000ml

    Minute CO2 emission – 1000ml

    Respiratory Quotient (RQ) = vCO2 /vO2 = 1000/1000(ml) = 1

   Carbohydrate = 1; Fat = 0.7; Protein = 0.8

5. A sportsman spontaneously held breath for 40 seconds, which resulted in an increase in heart rate and systemic arterial pressure. Changes of these indicators are due to activation of the following regulatory mechanisms:

Explanation

Reflex activity is the response to a peripheral nervous stimulation that occurs without our consciousness. It can be:

* Inborn or unconditioned: present from birth, does not require previous learning or training.

* Acquired or conditioned: developed after conditioning or training. Acquired after birth.

 

It is unconditioned because the sportman did not learn it; it is natural. The symptoms like increase in heart rate (HR) & systemic arterial pressure (AP) are sympathetic. Therefore, UNCONDITIONED SYMPATHETIC REFLEX

6. A 53-year-old male patient is diagnosed with Paget’s disease. The concentration of oxyproline in daily urine is sharply increased, which primarily means intensified disintegration of:

Explanation

Collagen, most abundant protein in human body; organizes and strengthens extracellular matrix. Collagen contains Gly-X-Y (X and Y are proline or lysine). Glycine(Gly) makes 1/3 of collagen. Oxyproline (hydroxyproline) is a major collagen amino acid which enables it to be regarded as a marker that reflects the catabolism of collagen.

 

Paget disease of bone (osteitis deformans): localized disorder of bone remodeling caused by increase in both osteoclastic and osteoblastic activity.

7. A patient has hoarseness of voice. During laryngoscopy a gray-white larynx tumor with papillary surface has been detected. Microscopic investigation has shown the following: growth of connective tissue covered with multi-layer, strongly keratinized pavement epithelium, no cellular atypia. What is the most likely diagnosis?

Explanation

Papillomas are benign epithelial neoplasms producing microscopically or macroscopically visible fingerlike or warty projections from epithelial surfaces. Polyp: when a neoplasm – benign or malignant produces a macroscopically visible projection above a mucosal surface and projects, for example, into the gastric or colonic lumen. Fibroma: benign, node of differentiated connective tissue. Angioma and angiofibroma are related to vessels.

8. During autopsy approximately 2,0 liters of pus have been found in the abdominal cavity of the corpse. Peritoneum is lustreless and has grayish shade, serous tunic of intestines has grayish-colored coating that is easily removable. Specify the most likely type of peritonitis in the patient:

Explanation

Fibrinopurulent peritonitis: Supurative or purulent inflammation is characterized by the production of large amounts of pus. The cellular inflammatory response in peritoneal cavity is composed primarily of dense collections of neutrophils and fibrinopurulent debris that coat the visceral and abdominal wall. Serous peritonitis – thin fluid (not pus); Hemorrhagic peritonitis  - hemorrhage; Tuberculous peritonitis – the patient must have TB, before there can be an extrapulmonary TB.

9. Autopsy of a dead patient revealed bone marrow hyperplasia of tubular and flat bones (pyoid marrow), splenomegaly (6 kg) and hepatomegaly (5 kg), enlargement of all lymph node groups. What disease are the identified changes typical for?

Explanation

IMG_9895

In chronic leukemia: spleen can weigh (6-8kg); liver (5-6kg). Chronic myelogenous leukemia (CML) is characterized by hepatosplenomegaly and generalized painless lymphadenopathy; pyoid bone marrow.

10. Autopsy of the dead patient who died from pulmonary edema revealed a large yellow-grey nidus in the myocardium, and a fresh thrombus in the coronary artery. What is the most likely diagnosis?

Explanation

The macro- and microscopic changes in the Myocardial Infarction (M.I.) correspond to the age of the infarct.

   * first 24hrs: No gross changes; coagulative necrosis

   * 1-3days: pallor of infracted tissue

   * 3-7days: red granulation tissue surrounds area of infarction

   * 7-10days: necrotic area is bright YELLOW

   * first 2 months: infarct is replaced by white, patchy, noncontractile scar tissue.

11. An animal experiment is aimed at studying the cardiac cycle. All the heart valves are closed. What phase of the cycle is characterized by this status?

Explanation

krushkrok No11 (2014)

Isometric or isovolumetric contraction period in cardiac cycle is the first phase of ventricular systole. There is increase in tension, without any change in length of muscle fibers. The volume does not change because all valves are closed.

Asynchronous contraction: myocardium is involved partly and the intraventricular pressure does not increase. Protodiastole is the first stage of ventricular diastole. Rapid filling – AV valves open, to admit blood into the ventricle. Reduced or slow filling – after the sudden rush of blood, the ventricular filling becomes slow.

12. Cyanide is a poison that causes instant death of the organism. What enzymes found in mitochondria are affected by cyanide?

Explanation

cyanide no 12

FullSizeRender (1)

Electron transport chain: NADH electrons from glycolysis enter mitochondria via the malate-aspartate or glycerol-3-phosphate shuttle. FADH2 electrons are transferred to complex II (at a lower energy level than NADH).

Inhibitors: complex I – Rotenone; complex III – Antimycin A; complex IV – cyanide, CO; complex V – Oligomycin. Cytochromes are electron carriers in the respiratory chain. They are arranged according to the values of their redox potentials in the following order: Cyt b; Cyt C1; Cyt C; Cyt a; Cyt aa3 . Cytochrome aa3 or cytochrome oxidase is a terminal enzyme that transfers electrons directly to O2.

13. Increased HDL levels decrease the risk of atherosclerosis. What is the mechanism of HDL anti-atherogenic action?

Explanation

krushkrok No13a (2014)FullSizeRender (9)

High density lipoprotein (HDL) – transports cholesterol from periphery to liver. Low Density Lipoprotein (LDL) - transports cholesterol from liver to tissues.

LCAT (lecithin-cholesterol acyltransferase) – catalyzes esterification of cholesterol

Cholesterol ester transfer protein (CETP) -  mediates transfer of cholesterol esters to other lipoprotein particles.

14. It has been found out that one of a pesticide components is sodium arsenate that blocks lipoic acid. Which enzyme activity is impaired by this pesticide?

Explanation

Pyruvate dehydrogenase complex: Mitochondrial enzyme complex linking glycolysis and citric acid cycle. The complex contains 3 enzymes that require 5 cofactors: Vit B1, B2, B3, B5, lipoic acid. Pyruvate → Acetyl-CoA.

 

The complex is similar to the α-ketoglutarate dehydrogenase complex (same cofactors, similar substrate and action), which converts α-ketoglutarate → Succinyl-CoA (CAC or TCA cycle). Arsenic inhibits lipoic acid. Findings: vomiting, rice-water stools, garlic breath.

15. Stool culture test revealed in a 6-month-old bottle-fed baby the strain of intestinal rod-shaped bacteria of antigen structure 0-111. What diagnosis can be made?

Explanation

Colienteritis - Antigen structure 0-111

The causative agents of colienteritis in children are O-groups -25; -26; -44; -55; -86; -91; -111; -114; -119; -125; -126; -127; -128; -141; -146 and others (they cause diseases in infants of the first month of life and in older infants). Most widely spread serotype is O-111:K58:H2

16. A boy referred to a genetics clinic was found to have 1 drumstick in blood neutrophils. The boy is likely to have the following syndrome:

Explanation

krushkrok No70 (2007)FullSizeRender (1)

Drumstick – barr body. Barr body is an inactive X-chromosome. So a boy (XY) with an inactive X-chromosome must have an additional X-chromosome – XXY (Klinefelter’s syndrome). Causes :

* nondisjunction (maternal and paternal nondisjunction in meiosis I)

* Mosaicism: with the karyotype being 46, XY/47, XXY

Manifestations: gynecomastia, female pattern of pubic hair distribution, no facial hair, high voice.

17. A drycleaner’s worker has been found to have hepatic steatosis. This pathology can be caused by the disruption of synthesis of the following substance:

Explanation

Hepatic steatosis can occur when humans are deprived of choline.

krushkrok No17 (2014)

Choline + Phosphatidic acid → Phosphatidylcholine (lecithin, PC). In the liver PC can also be synthesized from phosphatidylserine (PS) and phosphatidylethanolamine (PE), when free choline levels are low, because it exports significant amounts of PC in bile and as a component of serum lipoproteins (needed for fat metabolism)

PS → PE →→→ PC. 3 methylation reactions between PE and PC. S-adenosylmethionine is the methyl group donor. If choline, phosphatidylcholine or methionine is deficient, there will be abnormal phospholipid synthesis, oxidative damage caused by mitochondrial dysfunction, lipoprotein secretion (remember, if VLDL cannot be secreted it will be accumulated & cause fatty liver degeneration as seen in hepatic steatosis). PC is also a major lipid component of lung surfactant.

18. Ascarid eggs have been detected during stool analysis. What drug should be prescribed?

Explanation

Mebendazole – antihelminthic drug (chemotherapy of nematodes – roundworms). Ascarid eggs are gotten from Ascaris lumbricoides. Other drugs in this group are levamisolum, piperazinum,pyrantel. Nystatin is an antifungal; Chloramphenicol, Tetracycline, Furozolidone are antibiotics.
19. Bacteriological examination of the urine of the patient with pyelonephritis revealed microorganisms that produced yellow-green pigment and a characteristic odor in meat-peptone agar. What are they called?

Explanation

Pseudomonas produces blue green pigment; Yellow-green and red-brown pigments. Escherichia, proteas, Klebsiella, Azotobacter do not produce pigments.

20. Feces of a patient contain high amount of undissociated fats and have grayish-white color. Specify the cause of this phenomenon:

Explanation

image

image

Obturation (obstruction, to close) of bile duct – it can be:

* Intrahepatic – blockage of intrahepatic bile ducts

* Extrahepatic – blockage of common bile duct (ductus choledochus).

Findings: * malabsorption: bile salts do not enter the Small Intestine; no

                    emulsification of fat.

                  *light coloured stool: due to lack of urobilin (which leads to lack of

                   stercobilin).

                  *Jaundice (posthepatic, mechanic, obstructive): increased                                     conjugated

                   Bilirubin.

                 * Steatorrhea

The findings are specific for obstruction of bile duct and bile acid deficiency.

21. A 46-year-old female patient consulted a doctor about pain in the small joints of the upper and lower limbs. The joints are enlarged and shaped like thickened nodes. Serum test revealed an increase in urate concentration. This might be caused by a disorder in metabolism of:

Explanation

image

krushkrok No21 (2014)

This is a classic description of gout. The end product of the purine nucleotides catabolism in humans and other primates is uric acid (urate) which is excreted in urine. Allopurinol and febuxostat inhibits Xanthine oxidase (XO). Hypoxanthine and Xanthine which is more soluble is excreted in urine. Purine nucleotides (adenine and guanine).  AMP – Adenosine monophosphate; GMP – Guanosine monophosphate

Pyrimidine (thymine, Uracil, cytosine); catabolism of thymine and uracil gives urea, while catabolism of cytosine gives β-alanine.

22. A 26-year-old female consulted a doctor about having stool with white flat moving organisms resembling noodles. Laboratory analysis revealed proglottids with the following characteristics: long, narrow, with a longitudinal channel of the uterus with 17-35 lateral branches on each side. What kind of intestinal parasite was found?

Explanation

\"krushkrok

Taeniarhynchus saginatus (Taenia saginata) is a tapeworm. An intestinal parasite. It can be differentiated from other tapeworms by the number of uterine branches. Taenia saginata has 13 and more uterine branches, while other species such as Taenia solium have 5-12. Humans are infected by eating raw or undercooked beef containing larvae.

23. A male patient is 28 years old. Histological study of a cervical lymph node revealed a change of its pattern due to the proliferation of epithelioid, lymphoid cells and macrophages having nuclei in form of a horseshoe. In the center of some cell clusters there were non-structured light-pink areas with fragments of nuclei. What disease are these changes typical for?

Explanation

FullSizeRender (7)

When tubercle bacilli are introduced into the tissue, they are ingested by the alveolar macrophage. Macrophages undergo changes resembling epithelial cells – EPITHELOID cells. Some of the macrophages form MULTINUCLEATED GIANT cells by fusion of adjacent cells (langerhan’s or foreign body type). The giant cells may have 20 or more nuclei. These nuclei may be arranged at the periphery like HORSE-SHOE, RING or clustered at the poles or they may be present centrally (foreign body giant cells). Lymphocytes, plasma cells and fibroblasts surround the epitheloid cells and giant cells (hard tubercle- no central necrosis). Within 10-14 days, the centre of the cellular mass begins to undergo caseation necrosis – soft tubercle. This is the hallmark of tuberculous lesions.

24. A biochemical urine analysis has been performed for a patient with progressive muscular dystrophy. In the given case muscle disease can be confirmed by the high content of the following substance in urine:

Explanation

Creatine kinase/Creatine phosphokinase is an enzyme present in striated muscles, used to generate energy. When its serum level is elevated it is often an indication of muscle damage caused by injury, muscular dystrophy or cardiac problems. Most forms of muscular dystrophy are associated with decreased creatinine excretion. Creatinine is a break down product of creatine. Normal constituent of urine but can be elevated in muscular dystrophy. Since creatine is not a normal constituent of urine, it is more specific in this case.
25. While examining foot blood supply a doctor checks the pulsation of a large artery running in the separate fibrous channel in front of articulatio talocruralis between the tendons of long extensor muscles of hallux and toes. What artery is it?

Explanation

krushkrok No25 (2014) Pulses on the feet: dorsalis pedis pulse is found on the midanterior foot. Posterior tibial pulse – posterior to the medial malleolus. The anterior tibial artery passes through an opening in the interosseous membrane medial to the fibula and then becomes dorsalis pedis artery at the ankle/talocrural joint. Pulses on the feet: dorsalis pedis pulse is found on the midanterior foot. Posterior tibial pulse – posterior to the medial malleolus. The anterior tibial artery passes through an opening in the interosseous membrane medial to the fibula and then becomes dorsalis pedis artery at the ankle/talocrural joint.  
26. A patient with chronic heart failure with edema has increased level of blood aldosterone. What diuretic would be most effective in this case?

Explanation

IMG_9896

K+ sparing diuretics:

  - aldosterone antagonist: spironolactone, eplerenone

  - inhibit Na+ reabsorption: triamterene

     Carbonic anhydrase inhibitor: acetazolamide; Thiazide diuretics: hydrochlorthiazide; Loop diuretics: furosemide
27. During autopsy the following has been revealed: the meninges of the upper cerebral hemispheres are extremely plethoric, of yellow-green color and are soaked with purulent effluent. What kind of meningitis is characterised by such clinical presentations?

Explanation

Meningococcal meningitis is characterized by hyperemia of pia mater (extremely plethoric), by the end of the 2nd – 3rd day the exudate becomes thicker, green-yellow, purulent (yellow-green cap). Dull-serous exudates during the first days of the disease.

28. A 41-year-old male patient has a history of recurrent attacks of heartbeats (paroxysms), profuse sweating, headaches. Examination revealed hypertension, hyperglycemia, increased basal metabolic rate, and tachycardia. These clinical presentations are typical for the following adrenal pathology:

Explanation

Adrenal medulla produces catecholamines (epinephrine, norepinephrine and dopamine). Increased secretion of epinephrine & norepinephrine can result in these symptoms listed. Since the patient is 41yrs old, this can be pheochromocytoma - if it is a tumor.
29. A comminuted fracture of infraglenoid tubercle caused by shoulder joint injury has been detected during X-ray examination of a patient. What muscle tendon attached at this site has been damaged?

Explanation

krushkrok No29a (2014)krushkrok No29b (2014)

Origin:

Long head of triceps: infraglenoid tubercle of scapula

Long head of biceps: supraglenoid tubercle of scapula

Medial head of triceps: posterior surface of humerus, distal(inferior) to radial groove.

Lateral head of triceps: posterior surface of humerus, proximal(superior) to radial groove.

Short head of biceps: coracoid process of scapula

30. A patient has increased thickness of alveolar-capillary membrane caused by a pathologic process. The direct consequence will be reduction of the following value:

Explanation

Alveolar capillary membrane (respiratory membrane): formed by epithelium of respiratory unit and endothelium of pulmonary capillary. Exchange of gases between blood & alveoli takes place through respiratory membrane & this exchange occurs through bulk flow diffusion.

31. What drug will be most appropriate for the patient who has chronic gastritis with increased secretion?

Explanation

Pirenzepine (selective M1-cholinoblocker) – decrease gastric secretion. Pancreatine (polyenzyme drug) – increase gastric secretion. Aprotinin (contrycal, gordox) – antitrypsin; antiprotease. Chlorphentermine (desopimonum): appetite suppressant, stimulating CNS.

32. As a result of careless handling of an iron, a 34-year-old female patient has got acute pain, redness, swelling of her right index finger. A few minutes later, there appeared a blister filled with a transparent liquid of straw-yellow color. The described changes are a manifestation of the following pathological process:

Explanation

Inflammation consists of 3 stages:

* Alteration: characterized by injury of structurally-functional unit of an organ. Primary & secondary.

* Exudative: violation of microcirculation with exudation. Serous, fibrinous, purulent, decaying, hemorrhagic, combination. Serous – skin blisters, pericarditis with clear/transparent liquid(exudative inflammation).

 

* Proliferative: a phase of reparatory regeneration.

33. A 42-year-old male with a lesion of the ulnar nerve is unable to flex the II and V fingers to the midline. Which muscle function is impaired in this case?

Explanation

krushkrok No33 (2014)

Palmar interosseous muscles: adduction of the digits to the axial line/midfinger (finger III)

Dorsal interosseous muscles: abduction of the digits from the axial line.

34. A smear from the tonsillar coating of a patient with suspected diphtheria was found to contain blue bacilli with a thickening at the poles. What method of smear staining was used?

Explanation

Diphtheria bacteria (Corynebacterium diphtheria) is Gram positive, pleomorphic, often club-shaped rods and are arranged in palisades or in V (at an angle) or L-shaped formations. Media used for isolation are Tellurite agar & Lὄffler medium. Lὄffler nutrient medium consists of coagulated serum & nutrient broth. Selective indicator medium containing tellurite are used in selective culturing. K tellurite is used to inhibit the accompanying flora.

35. A child was born asphyxiated. What drug must be administered to the newborn to stimulate breathing?

Explanation

Analeptics (aethimizolum): drugs which act upon the CNS general functions, but stimulate mainly medulla oblongata (respiratory, cardiovascular centers). Aethimizolum acts on the medulla with a direct action.

 

Lobeline (N-cholinomimetic) can also act as an analeptic but with a reflexive type of action and does not influence the medulla directly. Prazosin (α1-receptor blocker); Atropine (M-cholinoblocker); Proserin/Neostigmine (cholinomimetic).

36. A 40-year-old patient with the progressing staphylococcal purulent periodontitis developed purulent inflammation of bone marrow spaces of the alveolar process, and then of the body of mandible. Microscopy revealed thinning of bone trabeculae, foci of necrosis, bone sequesters surrounded by the connective tissue capsule. What is the most likely diagnosis?

Explanation

Chronic osteomyelitis: most common causative agent is staphylococcus aureus. Characterized by bone necrosis (sequestrum formation), low-grade inflammation, persistent and progressing infection.  Also characterized by low-grade clinical presentations.
37. Human red blood cells do not contain mitochondria. What is the main pathway for ATP production in these cells?

Explanation

Lactate, formed by the action of lactate dehydrogenase (converting pyruvate to lactate) is the final product of anaerobic glycolysis in eukaryotic cells. In organs or cells that are poorly vascularized and/or lack mitochondria, formation of lactate is the major fate of pyruvate as seen in lens, cornea of the eye, kidney medulla, testes, leukocytes and red blood cells.

 

Aerobic glycolysis progresses to citric acid cycle from pyruvate. The cycle occurs totally in the mitochondria.

38. A 40-year-old female patient has undergone thyroidectomy. Histological study of thyroid gland found the follicles to be of different size and contain foamy colloid, follicle epithelium is high and forms papillae, there is focal lymphocytic infiltration in stroma. Diagnose the thyroid gland disease:

Explanation

IMG_9897

Basedow’s disease (Grave’s disease, diffuse toxic goiter, primary hyperplasia): prismatic epithelium turns into cylindrical, epithelium proliferation with formation of papillae, colloid vacuolization (foamy colloid), formation of lymphoid follicles with germ centers are observed.

Hashimoto’s thyroiditis is devoid of colloid.

39. A 43-year-old-patient has arterial hypertension caused by an increase in cardiac output and general peripheral resistance. Specify the variant of hemodynamic development of arterial hypertension in the given case:

Explanation

Eukinetic: Increase in both systolic and diastolic blood pressure.

Hyperkinetic: Increase in systolic blood pressure.

Hypokinetic: Increase in diastolic blood pressure.

Cardiac output determines systolic blood pressure; peripheral resistance determines diastolic blood pressure.
40. A patient has been hospitalised with provisional diagnosis of virus B hepatitis. Serological reaction based on complementation of antigen with antibody chemically bound to peroxidase or alkaline phosphatase has been used for disease diagnostics. What is the name of the applied serological reaction?

Explanation

Immune-enzyme analysis (enzyme-linked immunosorbent assays): used to detect the presence of either a specific antigen or a specific antibody in a patient’s blood sample. Common enzymes used are peroxidase, alkaline phosphatase or glucose oxidase. Producing an observable colour change or fluorescence. When radioactive isotopes are incorporated – Radioimmunoassay.

41. A patient has been hospitalised with provisional diagnosis of botulism. What serological reaction should be used to reveal botulinum toxin?

Explanation

Serological reactions or tests are carried out on a sample of blood serum which is used to detect serum antibodies or antibody-like substances that appear specifically in association with certain diseases. Types:

* Neutralization reaction: depend on capacity of antibody to neutralize infectious properties of infectious organisms. If the antibody is present to botulinum toxin, it confirms the diagnosis. It shows that the body has developed antibodies against the antigen (botulinum toxin). A patient who is not infected will not have the antibody.

* Precipitation reaction: takes place when the antibody and specifically prepared antigens are mixed together.

 

* Bordet-Gengou: specific for Bordetella pertussis.

42. Examination of a 52-year-old female patient has revealed a decrease in the amount of red blood cells and an increase in free hemoglobin in the blood plasma (hemoglobinemia). Color index is 0,85. What type of anemia is being observed in the patient?

Explanation

Increase in free hemoglobin in blood plasma is characteristic for intravascular hemolysis. It is definitely acquired since patient is 52years old. If it has an Hereditary cause, it will probably have an early onset.

43. The processes of heat transfer in a naked person at room temperature have been studied. It was revealed that under these conditions the greatest amount of heat is transferred by:

Explanation

Heat Radiation is a way the surface of the human body emits heat to the environment in the form of infrared rays. The amount of heat the body radiates to the environment is proportional to the surface of radiation area and to the difference between the mean values of skin and environment temperature. The surface radiation area is the total surface area of body parts that contact the air. Elimination of heat by radiation increases with a decrease in ambient temperature and decreases with its increase. It is possible to reduce elimination of heat by radiation via reduction of the surface of radiation area (“winding oneself into a ball”). Heat radiation does not require a medium for transfer of heat. (Key words: naked or lightly clothed).

Convection is a way the body eliminates heat by means of transferring heat via moving particles of air or water. To dissipate heat by means of convection, body surface shall be airflowed at a temperature that is lower than the temperature of the skin. At that, air layer contacting with the skin warms up, decreases its density, rises and is replaced by cooler, denser air. By increasing the speed of the air flow (wind, ventilation) heat emission increases significantly as well (forced convection). Convection requires convection current; current of gases or liquids (Key words: air over exposed area of skin).

Evaporation is a way the body dissipates heat to the environment by its evaporation via sweat or evaporation of moisture from the skin and respiratory tract mucous membranes of (“wet” heat loss). Evaporation closely related to relative humidity.

 

Conduction is a way the body eliminates heat by means of direct contact with another object. Heat is transferred down the temperature gradient (i.e. from the object of higher temperature to the object of lower temperature). Conduction requires contact with another object (Key words:  in water). 

44. Urine analysis has shown high levels of protein and erythrocytes in urine. This can be caused by the following:

Explanation

When blood passes through glomerular capillaries, the plasma is filtered. All substances of plasma are filtered except the plasma proteins and plasma cells, due to their large molecular size which is larger than the slit pores present in the endothelium of capillaries. Glomerular capillary membrane, basement membrane and visceral layer of bowman capsule form the filtration membrane through which glomerular filtration occurs. So, if protein and erythrocytes are increased or present in urine, it has to do with increase in renal filter permeability.
45. The development of both immune and allergic reactions is based upon the same mechanisms of the immune system response to an antigen. What is the main difference between the immune and allergic reactions?

Explanation

Immune reactions are directed against antigens. Allergic reactions (hypersensitivity) destroys tissue while trying to destroy the antigen leading to development of tissue lesion (tissue lesion – tissue injury). But both of them have the same mechanism.
46. After a craniocerebral injury a patient is unable to recognize objects by touch. What part of brain has been damaged?

Explanation

 krushkrok No46a (2014)IMG_9921

Postcentral gyrus (i.e. part of parietal lobe immediately posterior to central sulcus) is concerned with sensory information (touch).

Precentral gyrus (i.e. part of frontal lobe immediately anterior to central sulcus) is concerned with motor information. Occipital lobe: visual cortex. Cerebellum: coordination of movement and postural adjustment. Temporal lobe: superior temporal gyrus – located in temporal lobe immediately below sylvian fissure concerned with auditory stimuli. Hippocampus - memory

47. A patient complaining of rapid pulse, dyspnea and bluish color of mucosa has been admitted to the cardiological department. The objective symptoms are as follows: edema of lower extremities, ascites. Which of the given medicines should be prescribed for intravenous administration to improve the patient’s general state?

Explanation

The symptoms describe cardiac insufficiency. Cardiac glycoside is the most appropriate in this case. Cardiac glycosides are glycosidic drugs of plant origin having cardiac inotropic property, increase myocardial contractility and output. Corglyconum is short acting for acute cardiac insufficiency and administered intravenously.

Digitoxin is long-acting but basically administered orally and used mainly for chronic heart failure (also digoxin).

 

Cordiamin – analeptic; Adrenaline is commonly used in shocks and coma; Drotaverine – phosphodiesterase 4 inhibitor, antispasmodic (relaxant). Phosphodiesterase 3 inhibitor (nonglycoside cardiotonic)

48. As a result of a continuous chronic encephalopathy, a patient has developed spontaneous motions and a disorder of torso muscle tone. These are the symptoms of the disorder of the following conduction tract:

Explanation

Rubrospinal tract: red nucleus – lateral white column (spinal cord); controls muscle tone, because of its connections with cerebellum, vestibular apparatus and skeletal muscle.

Corticospinal tract: anterior and lateral – control of voluntary movements. Forms upper motor neuron (UMN).

 Corticonuclear tract: innervates muscles of the face, head and neck. Spinothalamic tract: anterior – crude touch sensation; lateral – pain and temperature sensations.

 

Tectospinal tract: control of movement of head in response to visual and auditory impulse.

49. A 10-year-old child has painful swallowing, neck edema, temperature rise up to 39, 0oC, the whole body is covered with bright-red petechial rash. Back of the throat and tonsils are hyperemic, the tongue is crimson-colored. Tonsillar surface is covered with isolated grayish-colored necrosis nidi. What disease is it?

Explanation

Scarlet fever is an acute infectious disease accompanied by local inflammatory changes mainly in the pharynx and typical generalized rash.

* Local changes: primary scarlatnic affect/complex – catarrhal or necrotic tonsillitis. Catarrhal tonsillitis: hyperemia of pharynx (flaring pharynx or burning faucet). Necrotic tonsillitis: coagulative necrosis and ulceration.

 

* General changes are first of all RASH. The face is also involved, but usually a small area about the mouth (nasolabial triangle) remains relatively unaffected to produce a circumoral pallor. Tongue is beefy red/crimson red and glistening, strawberry tongue.

50. As a result of a craniocerebral injury, a patient has a decreased skin sensitivity. What area of the cerebral cortex is likely to be damaged?

Explanation

krushkrok No46a (2014)IMG_9921

Postcentral gyrus (i.e. part of parietal lobe immediately posterior to central sulcus) is concerned with sensory information (touch).

Precentral gyrus (i.e. part of frontal lobe immediately anterior to central sulcus) is concerned with motor information. Occipital lobe: visual cortex. Cerebellum: coordination of movement and postural adjustment. Temporal lobe: superior temporal gyrus – located in temporal lobe immediately below sylvian fissure concerned with auditory stimuli. Hippocampus - memory

51.

In a patient elevation of body temperature takes turns with drops down to normal levels during the day. The rise in temperature is observed periodically once in four days. Specify the type of temperature curve:

Explanation

krushkrok No50 (2014)

* Febris intermittens: characterized by regular alternation of brief attacks of fever (paroxysms) with feverless periods (apyrexia). Attacks occur every 3rd day, 2nd day or everyday. Increase temperature persists for several hours, drops to normal and then rises again.

* Febris continua/persistent: elevated temperature persists at a high level, difference between morning and evening temperature does not exceed 10C.

* Febris recurrens: characterized by longer periods of pyrexia than intermittent (5-6days). Question says every 4 days.

* Febris hectic: 3-50C (difference in temperature).

* Febris remittens: difference in temperature exceeds 10C but temperature never falls to normal.

52. Diabetic nephropathy with uremia has developed in a patient with pancreatic diabetes. The velocity of glomerular filtration is 9 ml/min. What mechanism of a decrease in glomerular filtration velocity and chronic renal failure development is most likely in the case of this patient?

Explanation

Diabetic nephropathy results in mesangial expansion, glomerular basement membrane thickening, glomerulosclerosis by nonenzymatic glycosylation of glomerular basement membrane → increase permeability, thickening. This basically destroys the nephron, thereby reducing the number of active/normal nephrons.

 

Diabetics will most likely increase and not decrease the arterial pressure. Does not cause obstruction. Tissue acidosis will not necessarily affect glomerular filtration.

53. A 40-year-old patient has ulcer perforation in the posterior wall of stomach. What anatomical structure will blood and stomach content leak to?

Explanation

* Omental bursa is a closed part of peritoneal cavity, lying posterior to the stomach and lesser omentum.

* Pregastric or Antegastrial: lies anterior to the stomach.

* Hepatic bursa: embraces the right lobe of liver, gallbladder, upper pole of right kidney and right suprarenal gland.

* Right lateral channel (right paracolic gutter/sulcus)

 

* Left lateral channel (left paracolic gutter/sulcus)

54. A patient is diagnosed with hereditary coagulopathy that is characterised by factor VIII deficiency. Specify the phase of blood clotting during which coagulation will be disrupted in the given case:

Explanation

55. Angiocardiography of a 60-year-old male patient revealed constriction of a vessel located in the left coronary sulcus of the heart. What is the pathological vessel called?

Explanation

krushkrok No55 (2014)

* Ramus circumflexus (circumflex coronary artery) branch of left anterior descending artery runs within – left coronary sulcus.

* Arteria coronaria dextra (right coronary artery) – right coronary sulcus.

* Ramus interventricularis posterior (posterior interventricular branch): runs along the sulcus of the same name; greatest branch of right coronary artery

* Ramus interventricularis anterior (anterior interventricular branch): runs along the sulcus of the same name.

* Vena cordis parva (small cardiac veins): right portion of coronary sinus.

56. Those organisms which in the process of evolution failed to develop protection from H2O2 can exist only in anaerobic conditions. Which of the following enzymes can break hydrogen peroxide down?

Explanation

Peroxidase is a large family of enzymes and its optimal substrate is hydrogen peroxide e.g glutathione peroxide.

Catalase is found in organisms exposed to oxygen. It decomposes hydrogen peroxide to water and oxygen. H2O2 → H2O + O2

·        Hydroxylase: adds hydroxyl group (-OH) onto a substrate.

·        Oxygenase: oxidizer

 

·        Oxidase: oxidation-reduction reactions.

57. A patient complains of pain in the right lateral abdomen. Palpation revealed a dense, immobile, tumor-like formation. A tumor is likely to be found in the following part of the digestive tube:

Explanation

imageimage

Anterolateral abdominal wall has 9 regions and 4 quadrants (RUQ, LUQ, RLQ, LLQ)

·        Right lateral abdominal region: Ascending colon (colon ascendens), right kidney, right ureter and loops of small intestine.

·        Umbilical region: Transverse colon (colon transversum), head of pancreas, duodenum (except superior part)

·        Left lateral abdominal region: Descending colon (colon descendens), left kidney, left ureter and loops of small intestine.

·        Left inguinal region: Sigmoid colon (colon sidmoideum), left ureter, left external iliac artery of artery and vein.

·        Right Inguinal region: Caecum, vermiform appendix, right ureter

58. A patient complaining of pain in the left shoulder-blade region has been diagnosed with miocardial infarction. What kind of pain does the patient have?

Explanation

* Radiating pain: perceived at a site adjacent to or away from the site of origin but in the same dermatome i.e. supplied by afferent nerve fibers of one dorsal root.

* Viscera pain: organs, poorly localized, diffuse

* Protopathic pain: poorly localized pain

* Epicritic pain: well localized pain

* Phantom pain: pain from non-existent body structures (amputated limbs)

 

Phantom – if a leg has been amputated, the cut end heals with scar formation. The cut ends of nerve fibers are merged within the scar. If the cut end of sensory fibers are stimulated during movement of thigh, the patient feels as if the sensation is originating from non-existent leg. Sometimes the patient feels pain in non-existent limb.

59. A patient has a critical impairment of protein, fat and hydrocarbon digestion. Most likely it has been caused by low secretion of the following digestive juice:

Explanation

Pancreatic juice contains enzymes that can digest all food substances. α – amylase: carbohydrates; Lipase: fat; Proteases: protein. All others do not have all the enzymes needed to digest all 3 major food substances.

60. A female patient has facial neuritis that has caused mimetic paralysis and hearing impairment. Hearing impairment results from the paralysis of the following muscle:

Explanation

krushkrok No60 (2014)

Mimic muscles on the face and the stapedius muscle are supplied by the facial nerve (CN VII). So, paralysis of this nerve can lead to this symptoms. Facial nerve disorders may also manifest as taste acuity loss or impaired secretion of the lacrimal and salivary glands. Other muscles listed are not directly involved in hearing, as compared to the stapedius muscle, which is connected to the stapes (ear ossicle).

61. A 22-year-old female student consulted a physician about fever up to 38oC, weakness, sore throat. Objectively: there is white coating of the tongue. What histological structures of the tongue are involved in the formation of this coating?

Explanation

krushkrok No89 (2011)

Lingual papillae cover the dorsal surface of the tongue anterior to the sulcus terminalis of the tongue. 4 types:

* Filiform papillae: smallest and most numerous. No taste buds, serve only a mechanical role, distributed over the entire anterior dorsal surface. They are the ones that will be coated since they are numerous and serve mechanical role.

* Fungiform papillae: mushroom shaped, have taste buds

* Circumvallate papillae: large, dome-shaped structures that reside in the mucosa just anterior to the sulcus terminalis . It divides the tongue into anterior 2/3 and posterior 1/3.  Human tongue has 8-12 of it. It has taste buds.

* Foliate papillae: occur on the lateral edge of the tongue.

Taste buds are present on fungiform, foliate and circumvallate papillae.

62. A patient has a traumatic injury of sternocleidomastoid muscle. This has resulted in a decrease in the following value:

Explanation

All muscles that elevate the rib cage are muscles of inspiration and those that depress the rib cage are muscles of expiration.

Muscles of inspiration:

·        Sternocleidomastoid: lift upward on the sternum.

·        Anterior serrati: lift many of the ribs.

·        Scalene: lift the first two ribs.

Muscles of expiration: Abdominal recti – pull down the lower ribs and other abdominal muscles also compress the abdominal contents upwards against the diaphragm and internal intercostals.

 

Normal respiration is accomplished by the movement of the diaphragm only.

63. The receptors under study provide transfer of information to the cortex without thalamic involvement. Specify these receptors:

Explanation

Olfactory receptors (1st order neuron) → Mitral cells (2nd order neuron) → Olfactory tract → Olfactory trigone, anterior perforated substance, septum pellucidum (3rd order neuron) → Uncus of parahippocampal gyrus.

The Olfactroy analyzer is one of the oldest ones so it features several fibers that take the shortest route to the Olfactory cortex i.e. do not relay within the thalamus.

Gustatory: through its 2nd order neuron from medial lemniscus → thalamus.

Visual: through its subcortical visual centers → pulvinar of thalamus

 

Tactile (skin analyzer); Auditory: through lateral lemniscus of inferior colliculi of tectal plate and medial geniculate bodies of metathalamus.

64. Prolonged fasting causes hypoglycemia which is amplified by alcohol consumption, as the following process is inhibited:

Explanation

FullSizeRender (5)

image

Ethanol metabolism ↑NADH/NAD+ ratio in liver, causing:

·        Pyruvate → Lactate (lactic acidosis).

· Pyruvate → Lactate ; Oxaloacetate → malate ; both reations increase NADH/NAD+ ratio.

Prevents gluconeogenesis which results in fasting hypoglycemia.

65. A 39-year-old female patient with a history of diabetes was hospitalized in a precomatose state for diabetic ketoacidosis. This condition had been caused by an increase in the following metabolite level:

Explanation

\"krushkrok

In the liver, fatty acids and amino acids are metabolized to acetoacetate and β-hydroxybutyrate (to be used in muscle and brain). In prolonged starvation and diabetic ketoacidosis, oxaloacetate is depleted for gluconeogenesis. This cause a buildup of acetyl-CoA, which shunts glucose and free fatty acids (FFA) toward the production of ketone bodies. Breath smells like acetone (fruity odor). Urine test for ketones does not detect β-hydroxybutyrate.

66. A 37-year-old female patient complains of headache, vertigo, troubled sleep, numbness of limbs. For the last 6 years she has been working at the gas-discharge lamp-producing factory in the lead-processing shop. Blood test findings: low hemoglobin and RBC level, serum iron concentration exceeds the norm by several times. Specify the type of anemia:

Explanation

FullSizeRender (6)

Lead poisoning: lead inhibits Ferrochelatase and ALA dehydratase. Ferrochelatase catalyzes Protoporphyrin + Fe+ → Heme

ALA dehydratase catalyzes Aminolevulinic acid (ALA) → Porphobilinogen

Therefore, Iron, Protoporphyrin and ALA will be accumulated in the body. Iron refractory anemia results from disorder of Iron inclusion in heme at decrease of enzymes activity, which catalyzes synthesis of heme.

67. During an animal experiment, surgical damage of certain brain structures has caused deep prolonged sleep. What structure is most likely to cause such condition, if damaged?

Explanation

Reticular formation is a diffused mass of neurons and nerve fibers which form an ill-defined meshwork of reticulum in central portion of the brainstem. Functions: Ascending reticular activating system (ARAS) and descending reticular system. The ARAS is concerned with arousal, alertness, attention and wakefulness, emotional reactions, learning processes and conditioned reflexes. Hence, tumor or lesion in ARAS leads to prolonged sleeping or coma.

68. A patient takes cholagogues. What other process besides biliary excretion will be stimulated?

Explanation

Cholagolics are bile expelling drugs. Their pharmacological effects include: promotes bile excretion and emulsification of fats; absorption of fat and lipid soluble vitamins in intestine, enhance intestinal motor function (intestinal motility).

69. A 3-year-old child with meningeal symptoms died. Postmortem macroscopy of the pia matter revealed miliary nodules which were microscopically represented by a focus of caseous necrosis with masses of epithelioid and lymphoid cells with some crescent-shaped large cells in between having peripheral nuclei. Specify the type of meningitis in the child:

Explanation

FullSizeRender (9) FullSizeRender (7) When tubercle bacilli are introduced into the tissue, they are ingested by the alveolar macrophage. Macrophages undergo changes resembling epithelial cells – EPITHELOID cells. Some of the macrophages form MULTINUCLEATED GIANT cells by fusion of adjacent cells (langerhan’s or foreign body type). The giant cells may have 20 or more nuclei. These nuclei may be arranged at the periphery like HORSE-SHOE, RING or clustered at the poles or they may be present centrally (foreign body giant cells). Lymphocytes, plasma cells and fibroblasts surround the epitheloid cells and giant cells (hard tubercle- no central necrosis). Within 10-14 days, the centre of the cellular mass begins to undergo caseation necrosis – soft tubercle. This is the hallmark of tuberculous lesions.

During the hematogenous spreading, the most serious immediate complication is MILIARY tuberculosis. The name miliary derives from its resemblance to millet seeds. Lesions of miliary tuberculosis consist of small granulomas, with a central necrotic portion. Organs often affected are lung, spleen, liver, kidney, meninges and bone marrow.

70. A patient with homogentisuria has signs of arthritis, ochronosis. In this case, the pain in the joints is associated with the deposition of:

Explanation

image

Ochronosis (Alkaptonuria): congenital deficiency of homogentisate oxidase (homogentisic acid oxidase) in the degradative pathway of tyrosine to Fumarate → pigment-forming homogentisic acid accumulates (homogentisuria) in tissues. Autosomal recessive. Usually benign. Urine turns black on prolonged exposure to air. May have debilitating arthralgias (homogentisic acid toxic to cartilage).

71. A patient with hereditary hyperammonemia due to a disorder of ornithine cycle has developed secondary orotaciduria. The increased synthesis of orotic acid is caused by an increase in the following metabolite of ornithine cycle:

Explanation

image

FullSizeRender (10)

This is classic ornithine transcarbamylase deficiency. Most common urea cycle disorder. X-linked recessive. Interferes with the body’s ability to eliminate NH3. Excess Carbamoyl phosphate is converted to orotic acid (part of the pyrimidine synthesis pathway). Findings: ↑Orotic acid in blood and urine, symptoms of hyperammonemia. No megaloblastic anemia as seen in orotic aciduria – that’s why it is called secondary orotiic aciduria as used in the question.

Carbamoyl phosphate + Ornithine → Citrulline (urea cycle).

Carbamoyl phosphate is involved in 2 metabolic pathways: de novo pyrimidine synthesis and urea cycle. So, if the enzyme is deficient in urea cycle, all Carbamoyl phosphate will be channeled to de novo pyrimidine synthesis producing excess orotic acid.

72. Amniocentesis revealed two sex chromatin bodies (Barr bodies) in each cell of the sample. What disease is this character typical for?

Explanation

image

Barr body is an inactive X-chromosome. A normal female has one barr body XX, a normal male has no barr body XY.

Trisomy XXXX (only one X is active in a female; therefore, 2 barr bodies)

KlinefelterXXY (one barr body)

Turner’s – XO (no barr body)

Down and Patau involves autosomal chromosomes and not sex chromosomes.

73. A 49-year-old male patient with acute pancreatitis was likely to develop pancreatic necrosis, while active pancreatic proteases were absorbed into the blood stream and tissue proteins broke up. What protective factors of the body can inhibit these processes?

Explanation

α2-macroglobulin and α1-antitrypsin are protease inhibitors which can inhibit the pancreatic proteases and prevent further tissue protein breakdown. Immunoglobulin, Cryoglobulin – antibody; Ceruloplasmin – Copper; Transferrin – Iron; Haptoglobulin – hemoglobin; Interferon – released by cells in response to pathogens.

74. A 16-year-old female patient has fainted after quickly changing her body position from horizontal to vertical one. Which process from the ones listed below has caused the loss of consciousness in the first place?

Explanation

From horizontal (lying position) to vertical (standing position) will make all blood flow towards the lower extremities, thereby reducing venous return (blood going upwards towards the heart) in the first place. This will definitely ↓ cardiac output (C.O) → ↓blood supply to brain (this can lead to fainting). Then there is reflex vasoconstriction to increase venous return to heart, in the absence of any pathology.

75. A 53-year-old male patient complains of acute pain in the right hypochondrium. Objective examination revealed scleral icterus. Laboratory tests revealed increased ALT activity, and stercobilin was not detected in the stool. What disease is characterized by these symptoms?

Explanation

image

Obturation (obstruction, to close) of bile duct – it can be:

* Intrahepatic – blockage of intrahepatic bile ducts

* Extrahepatic – blockage of common bile duct (ductus choledochus).

Findings:

* malabsorption: bile salts do not enter the Small Intestine; no emulsification of fat.

*light coloured stool: due to lack of urobilin (which leads to lack of stercobilin).

*Jaundice (posthepatic, mechanic, obstructive): increased conjugated Bilirubin.

* Steatorrhea

The findings are specific for obstruction of bile duct and bile acid deficiency.

Liver is found in the right hypochondrium; sclera icterus is specific for jaundice; Absence of stercobilin in stool – blockage of bile duct. These symptoms indicate obstructive , mechanic, posthepatic jaundice. Cholelithiasis – gallstones which can obstruct bile flow → absence of stercobilin. Hemolytic jaundice → presence of stercobilin in stool.

76. A child patient has dry cough. What non-narcotic antitussive drug will relieve the patient’s condition?

Explanation

Narcotic – Opoid. Tussive – Cough.

Glaucine hydrochloride is the only antitussive centrally acting but non-opoid (non-narcotic).

 

Codeine phosphate and Morphine hydrochloride - antitussive centrally acting but opoid.

77. A patient complains of acute pain attacks in the right lumbar region. During examination the nephrolithic obturation of the right ureter in the region between its abdominal and pelvic segments has been detected. What anatomical boundary exists between those two segments?

Explanation

krushkrok No77 (2014)

Linea terminalis = pectineal line (pubis) + Arcuate line + sacral promontory + superior margin of pubic symphysis. Boundary between the abdominal and pelvic cavity.

Linea semilunaris found on the lateral margin of rectus abdominis.

Linea arcuata: the region on the posterior layer, where aponeuroses end and continue into the anterior layer. Part of linea terminalis anteriorly.

78. When measuring total muscle action potential it was revealed that it was subject to the power-law relationship. The reason for this is that individual muscle fibers differ in:

Explanation

The larger the stimulus, the greater the depolarization or attempt to reach depolarization or threshold potential. The threshold value controls whether or not the incoming stimuli are sufficient enough to generate an action potential. The threshold potential is the critical level to which the membrane potential must be depolarized in order to initiate an action potential. Individual muscle fibers and tissues generally require different depolarization threshold potential for action potential to occur.

79. A patient has insufficient blood supply to the kidneys, which has caused the development of pressor effect due to the constriction of arterial resistance vessels. This is the result of the vessels being greately affected by the following substance:

Explanation

FullSizeRender (12)

↓blood supply → activates renin. Renin catalyzes the conversion of angiotensinogen to angiotensin I. Angiotensin converting enzyme (ACE) converts angiotensin I → angiotensin II. Angiotensin II causes:

- vasoconstriction → ↑BP

- vasopressin (ADH) → ↑H2O reabsorption → ↑plasma volume

- aldosterone → ↑Na+ and H2O reabsorption → ↑plasma volume → ↑BP

80. A patient is diagnosed with iron-deficiency sideroachrestic anemia, progression of which is characterised by skin hyperpigmentation, pigmentary cirrhosis, heart and pancreas affection. Iron level in the blood serum is increased. What disorder of iron metabolism causes this disease?

Explanation

Sideroachrestic or sideroblastic anemia: causes includes

 

Chronic alcoholism (most common cause); Vit. B6 deficiency; Lead poisoning; X-linked recessive disease. It is a defect in heme synthesis within the mitochondrial. Iron accumulates in mitochondria, producing ringed sideroblast. X-linked defect in δ-ALA synthase gene. It is classified as an Iron-overload type of anemia. This enzyme catalyzes Glycine + Succinyl-CoA → δ-ALA rate limiting step in heme synthesis. Iron cannot be used in heme synthesis since the process has been disturbed earlier with the deficiency of the enzyme, therefore, Iron is accumulated.

81. In a village, a case of anthrax had been registered. Medical services began epidemiologically indicated specific prophylaxis of population against anthrax. What preparation was used for this purpose?

Explanation

Live spore vaccine (STI) is used for vaccination against anthrax. STI live vaccine
82. Experimental stimulation of the sympathetic nerve branches that innervate the heart caused an increase in force of heart contractions because the membrane of typical cardiomyocytes permitted an increase in:

Explanation

IMG_9903

When a muscle is excited (stimulated) by the impulses passing through neuromuscular junction, action potential is generated which spreads over sarcolemma (plasma membrane of muscles). When the action potential reaches the cisternae of ‘L’ tubules, Ca2+ stored in the cisternae are released into the sarcoplasm (cytoplasm of muscles). The Ca2+ moves towards the actin filaments to produce the contraction. Therefore, it is Ca2+ entry into the sarcoplasm.

83. According to the results of glucose tolerance test, the patient has no disorder of carbohydrate tolerance. Despite that, glucose is detected in the patients’s urine (5 mmol/l). The patient has been diagnosed with renal diabetes. What renal changes cause glucosuria in this case?

Explanation

Food substances like glucose, amino acids, acetoacetate ions and vitamins are completely reabsorbed from renal tubules and do not appear in urine under normal conditions. These substances can appear in urine, only if their concentration in plasma is abnormally high (for glucose above 10mmol/L – renal threshold) or in renal diseases when reabsorption is affected. Since glucose is 5mmol/L, then it has to be a problem with decreased reabsorption of glucose.

84. As a result of past encephalitis, a male patient has developed an increase in cerebrospinal fluid pressure in the right lateral ventricle. What can be the cause of this condition?

Explanation

IMG_9904

FullSizeRender (15)

Encephalitis is inflammation of the brain. 4 ventricles containing cerebrospinal fluid (CSF) within the brain. CSF is formed by the choroid plexus, within the cerebral ventricles. CSF moves from the lateral ventricles (right and left) → interventricular foramen (of Monro) → 3rd ventricle → cerebral aqueduct (of sylvius) → 4th ventricle. An ↑ in CSF pressure in right lateral ventricle is due to obstruction or closure of the right interventrcular foramen preventing outflow of CSF.

85. A 3-year-old child has continuous fever, lymph nodes are enlarged, the amount of lymphocytes in blood is significantly increased. Enzyme-linked immunosorbent assay (ELISA) revealed antigen of Epstein-Barr virus. What diagnosis can be made based on the information given above?

Explanation

IMG_9905

Anti-Epstein-Barr nuclear antigen antibody is specific and highly sensitive test or laboratory finding for Infectious mononucleosis. Epstein Barr virus (EBV) is a herpes virus and causative agent of Infectious mononucleosis, also called kissing disease.

Associated with Burkitt lymphoma (a neoplasia but no neoplastic lymphocytes mentioned in the question).

86.

Parents of 5-year-old child report him of having frequent colds that develop into pneumonias, presence of purulent rashes on the skin. Laboratory tests have revealed the following: absence of immunoglobulins of any type, and naked cells are absent from the lymph nodes punctate. What kind of immune disorder is it?

Explanation

X-linked (Bruton) agammaglobulinemia: defect in Bruton’s tyrosine kinase (BTK), a tyrosine kinase gene → no B cell maturation. X-linked recessive (↑in boys). Findings: absent B cells in peripheral blood, ↓immunoglobulin of all classes. Absent/scanty lymph nodes and tonsils.
87. A microslide contains the preparation of a gland composed of several secretory saccule-shaped parts that open in the common excretory duct. What gland is it?

Explanation

88. A patient intending to undergo a gender reassignment surgery has been admitted to a specialised clinic. In the course of examination both male and female gonades have been revealed, with male structure of external genitals. What kind of genital maldevelopment has the patient?

Explanation

Hermaphroditism refers to ambiguous sexual differentiation due to development disorders during embryonic period. 2 types: true and false (pseudo). True hermaphroditism is a rare-type state featuring presence of both female and male gonads in an organism. Pseudohermaphroditism is more common and further divided into male and female. Male: male gonads, 46XY but external genitals are intersexual or resemble female. Female: female gonads, 46XX but external genitals resemble those in males.

89. An 18-year-old male has been diagnosed with Marfan syndrome. Examination revealed a developmental disorder of connective tissue and eye lens structure, abnormalities of the cardiovascular system, arachnodactylia. What genetic phenomenon has caused the development of this disease?

Explanation

* Pleiotropy: one gene contributes to multiple phenotypic effects → disorder of connective tissue and eye lens structure, abnormalities of CNS, arachnodactylia. Marfan is FBN1 gene mutation on chromosome 15; Marfan is one gene defect causing all this phenotypic defects.

* Codominance: both alleles contribute to the phenotype of the heterozygote. e.g. blood group AB

* Incomplete dominance: intermediate inheritance. Red + White → Pink

 

* Multiple allelism e.g. ABO blood type

90. In case of alkaptonuria, homogentisic acid is excreted in urine in large amounts. The development of this disease is associated with a disorder of metabolism of the following amino acid:

Explanation

image

Phenylalanine → Tyrosine →→→ Homogentisic acid → Maleylacetoacetic acid

91. A male patient complains of skin insensitivity of inferior eyelid, external lateral surface of nose and upper lip. A doctor in the course of examination has revealed inflammation of the second branch of trigeminal nerve. What cranial foramen does this branch go through?

Explanation

No40(2013)
92. A patient with insomnia induced by allergic rash and itch has been prescribed the drug that has antihistamine and hypnotic effect. Specify this drug:

Explanation

Benadryl (Diphenhydramine, Dimedrol): Antihistamine (1st generation) – H1 receptor blocker. Has sedative, hypnotic and ganglioblocking effects. Loratadine (2nd generation) – also  an antihistamine but has a weak sedative, hypnotic effect. Prednisolone, Acetylsalicylic acid and Analgin are not antihistamines.

93. In a cat with decerebrate rigidity the muscle tone is to be decreased. This can be achieved by:

Explanation

Decerebrate rigidity indicates brain stem damage, specifically damage below the level of the red nucleus. Vestibular nerve has 4 nuclei – superior(Bechterew’s), inferior(Roller), lateral(Deiter’s) and medial(Schwalbe’s). In the lateral vestibular nucleus, some fibers from this nucleus pass into the cerebellum, which is important in regulating muscle tone. It has been shown experimentally that destruction of this nucleus leads to loss of tone almost completely.

94. When studying the signs of pulmonary ventilation, reduction of forced expiratory volume has been detected. What is the likely cause of this phenomenon?

Explanation

 Forced expiratory volume (FEV) is the volume of air, which can be expired forcefully in a given unit of time (FEV1 - In 1 second). It is very much decreased in obstructive diseases like asthma and emphysema.

95. X-ray examination of a patient allowed to diagnose a tumor in the superior lobe of the right lung. There is a probability of metastases spread to the following lymph nodes:

Explanation

The dominant lymphatic drainage from the right upper lobe flowed into the superior mediastinal nodes. The sternal or parasternal nodes is one of the superior mediastinal nodes.
96. A 55-year-old patient with a characteristic rash, fever, dizziness has been admitted to a hospital. He has been provisionally diagnosed with typhus. No similar cases have been reported. In his youth (15 years old) the patient suffered typhus in a boarding school. What disease is it?

Explanation

Epidemic typhus (human body louse) – Rickettsia prowazekii, can remain latent and reactivate months or years in an infected patient, with symptoms similar to or even identical to the original attack of typhus, including a rash that starts centrally and spreads out, sparing palms and soles. This delayed relapse of epidemic typhus – Brill’s disease.

97. A young woman suddenly developed fever up to 39oC accompanied by a strong headache. Examination revealed marked nuchal rigidity. Spinal puncture was performed. Gram-stained smear of cerebrospinal fluid contained many neutrophils and Gram-positive diplococci. What bacteria could be the cause of this disease?

Explanation

Gram negative diplococcic are Neisseria meningitides and N. gonorrhoeae.

N. meniingitidis cause meningitis, meningococcemia (nuchal rigidity is a sign of meningitis).

 

N. gonorrhoeae causes gonorrhea, septic arthritis. Sexually transmitted or perinatally.

98. In the course of an experiment adenohypophysis of an animal has been removed. The resulting atrophy of thyroid gland and adrenal cortex has been caused by deficiency of the following hormone:

Explanation

6 hormones are secreted by anterior pituitary/adenohypophysis: GH; TSH; ACTH; FSH; LH and Prolactin. They are responsible for the growth and secretory activity of their target glands. The first 5 hormones (Prolactin not included) are called tropic hormones.

99. Degenerative changes in posterior and lateral columns of spinal cord (funicular myelosis) caused by methylmalonic acid accumulation occur in patients with B12-deficiency anemia. This results in synthesis disruption of the following substance:

Explanation

IMG_9833

Funicular myelosis: Vit B12 deficiency – subacute combined degeneration – demyelination of dorsal columns (posterior), lateral corticospinal tracts and spinocerebellar tracts.

 

Fatty acids →→ Methylmalonyl-CoA → Succinyl-CoA → TCA cycle, myelin synthesis, Heme

100. During local anesthetization the patient has gone into anaphylactic shock. What drug must be administered to the patient?

Explanation

*Epinephrine/Adrenaline hydrochloride: is an α-β-adrenergic agonists (adrenomimetics), direct acting. Uses: Anaphylactic shock, hypoglycemic coma…

* Diazepam: Tranquilizer (Benzodiazepine). Uses: anxiety, status epilepticus, night terrors.

* Atropine sulphate: M-cholinoblocker. Uses: eye traumas, GI diseases with ↑acidity.

* Propanolol: β-blocker. Uses: hypertension, Ischemic heart disease.

 

* Nitroglycerine: Organic nitrates. Uses: angina attack

101. As a result of an injury, the integrity of the anterior spinal cord root was broken. Specify the neurons and their processes that had been damaged:

Explanation

FullSizeRender (16)

Anterior (ventral, motor) spinal cord root: arises from the anterolateral sulcus and contains a set of axons of motor neurons located within the anterior columns; the anterior roots number 31 pairs. The roots of the segments C8 through L2 also comprise the autonomic (sympathetic) fibers that arise from the sympathetic nuclei of the lateral grey columns.

102. A patient who had been taking diclofenac sodium for arthritis of mandibular joint developed an acute condition of gastric ulcer. Such side effect of this medicine is caused by inhibition of the following enzyme:

Explanation

Diclofenac-natrium(sodium) is a non-selective inhibitor of COX-1 and COX-2. Cyclooxygenase (COX) converts arachidonic acid into the endoperoxide precursors of prostaglandin. COX-1 is present in non-inflammatory cells (normal body tissues); COX-2 is present in lymphocytes, polymorphonuclear cells and other inflammatory cells. Side effects: GI symptoms – bleeding, ulceration and perforation. Because COX-1 is used to produce mediators that help to protect gastric mucosa, whenever COX inhibitors are used they produce this side effects.

103. In one of Polessye regions there was an outbreak of helminthiasis manifested by cramps and facial edemata. The developed preventive measures in particular included ban for eating infested pork even after heat processing. What helminthiasis was the case?

Explanation

Trichinosis: causative agent – Trichinella spiralis. Transmission – fecal-oral; undercooked meat (especially pork). Findings: fever, vomiting, nausea, periorbital edema, myalgia. These findings are specific for Trichinosis.

104. A female patient saught medical-genetic consultation. Physical examination revealed pterygium colli deformity (webbed neck), broad chest, underdeveloped breasts. Study of buccal epithelium cells revealed no X-chromatin in the nuclei. This indicates that the patient has the following syndrome:

Explanation

krushkrok No105 (2014)

FullSizeRender (1)

These characterize Turner’s syndrome (XO). It can be complete monosomy (45,XO) or mosaicism (e.g. 45,XO/46,XX). Klinefelter is relate dto males. The rest are not related to sex chromosomes, but they are autosomoes.

105. During blood transfusion a patient has developed intravascular erythrocyte hemolysis. What kind of hypersensitivity does the patient have?

Explanation

FullSizeRender (17)

* Type I: Allergic and atopic disorders (e.g. rhinitis, hay fever, asthma); Anaphylaxis. Presentation – immediate, anaphylactic, atopic

* Type II: Acute hemolytic transfusion reactions, autoimmune hemolytic anemia, Erythroblastosis fetalis, Goodpasture syndrome, Graves disease, Myasthenia gravis. Disease tends to be specific to tissue or site where antigen is found.

* Type III: Arthus reaction; SLE; serum sickness. Can be associated with vasculitis and systemic manifestations.

* Type IV: Contact dermatitis, Graft-versus-host disease, multiple sclerosis. Response is delayed and does not involve antibodies (vs. Type I, II, III)

106. Patients with erythropoietic porphyria (Gunther’s disease) have teeth that fluoresce with bright red color when subjected to ultraviolet radiation; their skin is light-sensitive, urine is red-colored. What enzyme can cause this disease, when it is deficient?

Explanation

Erythropoietic porphyria (Gunther’s disease): This disorder is due to a defect in the enzyme Uroporphyrinogen III cosynthase. It is characterized by:

* It is a rare congenital disorder caused by autosomal recessive mode of inheritance, mostly confined to erythropoietic tissues.

* The individuals excrete Uroporphyrinogen I and Coproporphyrinogen I which oxidize respectively to Uroporphyrin I and Coproporphyrin I (red pigments).

* The patients are photosensitive (itching and burning of skin when exposed to visible light) due to the abnormal porphyrins that accumulate. Porphyrins are accumulated in the teeth, bones and an increases amount are seen in the plasma, bone marrow, faeces, RBCs and urine.

 

* Increased hemolysis is also observed in the individuals affected by this disorder.

107. During the air and bone conduction tests it was revealed that in the left ear the tones were louder by bone conduction. This might be associated with the disease of:

Explanation

krushkrok No105 (2007)

Normal hearing: air conduction should be greater than bone conduction. Air conduction uses the (pinna, eardrum and ossicles) to amplify and direct the sound whereas bone conduction bypasses some or all of these and allows the sound to be transmitted directly to the inner ear. In conductive hearing loss, bone conduction is better (louder) than air (negative Rinne test). This confirms the diagnosis that the left part of the ear has a problem with conduction of sound to the inner ear → left middle ear. In this case the inner ear is normal since the patient can still hear. It is therefore, a pathology of either the pinna (outer ear), eardrum or ossicles (middle part of ear).

108. In course of invasive abdominal surgery a surgeon has to locate the origin of the mesenteric root. Where is it normally localized?

Explanation

krushkrok No109 (2014) The mesentery fixes the intestine to the posterior abdominal wall; its posterior margin is short (15-20mm) and is called the root of mesentery directed obliquely from the duodenojejunal flexure at the left side of the second lumbar vertebra to the right of sacroiliac joint.
109. A 55-year-old male had been delivered to the resuscitation unit unconscious. Relatives reported him to have mistakenly drunk an alcoholic solution of unknown origin. On examination the patient was diagnosed with methanol intoxication. What antidote should be used in this case?

Explanation

FullSizeRender (5) The specific antidote for methanol or ethylene glycol poisoning/intoxication is ethanol or fomepizole by inhibiting alcohol dehydrogenase which delays the methanol metabolism until methanol is eliminated; Methanol is relatively nontoxic, but is metabolized by alcohol dehydrogenase to toxic metabolites.
110. Due to the use of poor-quality measles vaccine for preventive vaccination, a 1-year-old child has developed an autoimmune renal injury. The urine was found to contain macromolecular proteins. What process of urine formation has been disturbed?

Explanation

When blood passes through glomerular capillaries, the plasma is filtered. All substances of plasma are filtered except the plasma proteins and plasma cells, due to their large molecular size which is larger than the slit pores present in the endothelium of capillaries. Glomerular capillary membrane, basement membrane and visceral layer of bowman capsule form the filtration membrane through which glomerular filtration occurs. So, if protein and erythrocytes are increased or present in urine, it has to do with increase in renal filter permeability.

111. As a result of a road accident a 37-year-old female victim developed urinary incontinence. What segments of the spinal cord had been damaged?

Explanation

IMG_9908

Urinary incontinence – involuntary urination. In general, parasympathetic stimulation are anabolic, promoting normal function and conserving energy. The sacral part of parasympathetic system reside within the S2-S4 sacral segments of the spinal cord. They inhibits contraction of internal sphincter of bladder, contracts detrusor muscle of the bladder wall causing urination (voluntary). Lesion to this segment of spinal cord (S2-S4) causes involuntary urination.

112. An 8-week-pregnant woman with acute respiratory disease and temperature rise up to 39, 0oC has called in a doctor. The doctor insisted on her avoiding taking paracetamol, because in this period of pregnancy there is a risk of its:

Explanation

There is structural damage (i.e. affecting organ formation) in a 1st trimester exposure and functional damage (i.e. affecting organ maturation and/or function) in 2nd and 3rd trimester exposures to paracetamol. It is teratogenic because it is able to disturb the growth and development of an embryo or fetus.

113. A pneumonia patient has been administered acetylcysteine as a part of complex therapy. What principle of therapy has been taken into consideration when applying this drug?

Explanation

Acetylcysteine is an expectorant (mucolytic), which ↑bronchial secretion or reduce its viscosity, facilitating its removal by coughing; they loosen cough which becomes less tiring and more productive. This drugs treats the pathogenesis/development of the disease and not its cause (etiology-etiotropic) or symptoms (symptomatic).

114. A 26-year-old female patient with bronchitis has been administered a broad spectrum antibiotic as a causal treatment drug. Specify this drug:

Explanation

Doxycycline is a tetracycline, broad spectrum antibiotic. Bacteriostatic; bind to 30s subunit of ribosome. All other options are not antibiotics.

115. When defining blood group according to the AB0 system, using salt solutions of monoclonal antibodies, agglutination didn’t occur with any of the solutions. What blood group is it?

Explanation

Blood group O(I): no antigens, therefore no agglutination.

Blood group A(II): A antigen, agglutinate with blood group B(III) and O(I).

Blood group B(III): B antigen, agglutinate with blood group A(II) and O(I).

Blood group AB(IV): A and B antigen, agglutinate with all blood groups. No antibody.

 

Blood group name is determined by the antigen present on RBC, but the patient has an opposite antibody. So whenever, the antibody corresponds to the antigen, there is agglutination. Since O does not have any antigen, no agglutination can occur.

116. Examination of a 56-year-old female patient with a history of type 1 diabetes revealed a disorder of protein metabolism that is manifested by aminoacidemia in the laboratory blood test values, and clinically by the delayed wound healing and decreased synthesis of antibodies. Which of the following mechanisms causes the development of aminoacidemia?

Explanation

Type I diabetes: lack of insulin. Normally, insulin stimulates storage of lipids, proteins and glycogen. But in the absence of insulin, glucagon and epinephrine stimulates use fuel reserves through hepatic glycogenolysis, hepatic gluconeogenesis, adipose release of free fatty acids (FFA). ↑in glucagon and epinephrine → ↑protein degradation (muscle) → ↑amino acid → aminoacidemia. Protein degradation = proteolysis.

117. Symptoms of pellagra (vitamin PP deficiency) is particularly pronounced in patients with low protein diet, because nicotinamide precursor in humans is one of the essential amino acids, namely:

Explanation

Vit. B3 (Niacin, PP): coenzyme forms NAD+, NADP+ is derived from tryptophan. Synthesis requires Vit. B2 and B6.

118. A 68-year-old patient consults a cardiologist, complaining of high arterial blood pressure, pain in the heart region, intermittent pulse. Prescribe the β1-adrenoreceptor blocker for the treatment of the described pathology:

Explanation

β1-adrenoblocker (cardioselective) – Metoprolol: blocks β1-adrenoreceptors of the heart. Fenoterol β2-adrenomimetics (short-acting 4-6hrs). Benzylpenicillin – short acting penicillin (β-lactam antibiotic). Nootropil – Nootropic. Morphine hydrochloride – Narcotic analgesic.

119. A female suffered rubella during pregnancy. The child was born with developmental abnormalities, namely cleft lip and palate. The child’s genotype is normal. These malformations are a manifestation of:

Explanation

Variability represents the propriety of an organism to gain new traits, different from those of their parents. Variations may be induced by internal factors (hormones, metabolites) and external factors (physical, chemical and biological). The main biological factors are viruses (rubella virus), bacterial and fungal toxins. Types of variability: hereditary (genotypic) and Non-hereditary (phenotypic, modification). Non-hereditary/Modification variability: different factors (e.g. rubella virus in the question) may change the phenotype, without modifying the DNA i.e. does not affect the genotype. Hereditary on the other hand can be caused by mutations in the DNA.

120. An electron micrograph shows a cell-to-cell adhesion consisting, in each cell, of an attachment plaque. The intercellular space is filled with electron-dense substance including transmembrane fibrillar structures. Specify this adhesion:

Explanation

image

Anchoring junctions provide lateral adhesions between epithelial cells. 2 types: Zonula adherens and Macula adherens or Desmosome. In epidermal cells, the desmosome is the only attachment device present. On the cytoplasmic side of the plasma membrane of each of the adjoining cells is a disk shaped structure consisting of very dense material called the desmosomal attachment plaque. This attachment plaques anchors intermediate filaments. The intercellular space is occupied by electron-dense material containing desmocollins and desmogleins. Tight junctions – occluding junction.

121.

For biochemical diagnostics of myocardial infarction it is necessary to measure activity of a number of enzymes and their isoenzymes. What enzymatic test is considered to be the best to prove or disprove the diagnosis of infarction in the early period after the chest pain is detected?

Explanation

IMG_9910

In laboratory diagnosis of acute myocardial infarction: Creatine kinase isoenzyme MB (CK-MB) appears within 4-8hrs, peaks at 24hrs and disappears within 1.5-3 days. Sensitivity and specificity is 95%. Reappearance of CK-MB after 3days – reinfarction. CK-MM – found in skeletal muscle and heart (not specific for heart). LDH1 – heart muscle; LDH2 – blood serum. LDH levels are also high in tissue breakdown or hemolysis. Although CK-MB is more specific and sensitive for infarction than LDH. Aspartate transaminase is not specific for heart damage alone but can still be used to diagnose myocardial infarction. But troponin test [cardiac troponins I (cTnI) and T (cTnT)] is the most sensitive and specific test for myocardial infarction.

122.

A patient complains of pain in the heart area during acute attack of gastric ulcer. What vegetative reflex can cause this painful feeling?

Explanation

*Viscerovisceral reflex: functional or organic disease of a viscera (organ) causes pain, irritation to another organ e.g gastric ulcer (stomach) – heart

*Viscerodermal reflex: organ-skin

*Visceromotor reflex: organ-muscle

*Dermatovisceral reflex: skin-organ

*Motor-visceral reflex: muscle-organ

 

This is because the skin and related organs or muscle have the same segmental innervations usually by dorsal roots, spinal nerves and nuclei (referred pain resulting from reflex phenomena).

123.

A patient is diagnosed with acute morphine hydrochloride intoxication. Prescribe an oxidizing agent for gastric lavage:

Explanation

Potassium permanganate: KMnO4 →MnO2 + O2. MnO2 has astringent action. O2  oxidizes proteins in microbes, deodorizing action, antidote action. 0.1% - 100ml for gastric lavage. Chloramines and chlorhexidine bigluconate – nonorganic antiseptic and disinfectant  → chlorine containing (halogens) for treatment of wounds and processing of surgeon’s hands. Sulfocamphocainum – respiratory stimulant.

124.

In the course of puncture biopsy of transplanted kidney the following has been revealed: diffuse infiltration of stroma by lymphocytes and plasmocytes and necrotic arteritis. What pathological process has developed in the transplant?

Explanation

A biopsy of the transplanted organ can confirm that it is being rejected. The laboratory pathologist generally seeks 3 main histological signs:

* Infiltrating T cells, perhaps accompanied by infiltrating plasma cells, eosinophils and neutrophils.

* Structural compromise of tissue anatomy.

* Injury to blood vessels.

 

Rejection is an adaptive immune response. Therefore, immune rejection.

125.

During cell division, DNA replication occurs by a signal from the cytoplasm, and a certain portion of the DNA helix unwinds and splits into two individual strains. What enzyme facilitates this process?

Explanation

image

*Helicase unwinds DNA template at the replication fork.

*DNA-dependent RNA polymerase (primase) makes an RNA primer on which DNA polymerase III can initiate replication.

*Ligase seals fragments of newly synthesized DNA fragments (i.e. joins okazaki fragments).

*DNA polymerase III elongates leading strand and lagging strand. DNA polymerase I degrades RNA primer, replaces it with DNA.

126.

A female patient, having visited the factory premises with lots of dust in the air for the first time, has got cough and burning pain in the throat. What respiratory receptors, when irritated, cause this kind of reaction?

Explanation

Irritant receptors are located in the bronchi and bronchioles of lungs and are stimulated by irritant chemical agents such as ammonia, sulfur dioxide and some other small particles like dust, smoke etc. These receptors send afferent impulses to respiratory centers via vagal nerve fibers. Stimulation of irritant receptors produces reflex hyperventilation along with bronchospasm. J receptors are stimulated during pulmonary congestion, pulmonary edema, over-inflation of lungs. Stretch receptors are the receptors which give response to stretch of the tissues – Hering-Breuer reflex is initiated by the stimulation of stretch receptors of air passage. Proprioceptors of respiratory muscles are the receptors which give response to change in the position of body. Thermoreceptors are cutaneous receptors, which give response to change in the environmental temperature.

127.

Since a patient has had myocardial infarction, his atria and ventricles contract independently from each other with a frequency of 60-70 and 35-40 per minute. Specify the type of heart block in this case:

Explanation

Complete atrioventricular block (third degree heart block; complete heart block) is the condition in which the impulses produced by sinoatrial node (SA node) cannot reach the ventricles. Because of this, the ventricles beat in their own rhythm, independent of atrial beat. Partial or 2nd degree heart block: transmission of impulses produced from SA node fail to reach the ventricles. It iis slowed down and not blocked completely. Sinoatrial block is failure of impulse transmission from SA node to atrioventricular (AV) node. Intra-atrial block: transmission of impulse through atrial myocardium is impaired. Intraventricular block: conduction of impulse through His bundle and its branches is impaired.

128.

A 35-year-old female patient has been hospitalised with acute intoxication caused by salts of high-density metals (lead, most probably). As a part of complex therapy the antidote that contains two active sulfhydric groups has been prescribed. Specify this anti-dote:

Explanation

Dimercaprol (unithiol, British anti-lewisite) is used to chelate mercury, bismuth, arsenic and in combination with edentate calcium disodium to treat lead intoxication. It contains two sulfuhydryl (SH-) groups and forms two bonds with metal ions. Administered intramuscularly.

 

Mannitol – osmotic diuretic; Nalorphine hydrochloride – antidote, blocks opoid receptors and replace opoids from binding with them. Metamizole: analgesic, non-opoid drug. Calcium chloride – Calcium preparation, coagulant with systemic action.

129.

A 60-year-old male patient has type II diabetes. A doctor has prescribed him synthetic hypoglycemic long-acting drug that is sulfonylurea derivative. What drug is it?

Explanation

Sulfonylurea are insulin secretagogues because they promote insulin release from the β-cells of the pancreas. The primary drugs used today are the 2nd generation drugs – glibenclamide, glipizide, glimepiride etc. These agents are useful in the treatment of patients who have type 2 diabetes mellitus and cannot be managed by diet alone. Metformin is the only currently available biguanide; its classed as an insulin sensitizer. Acarbose – α glucosidase inhibitor. Butamide (tolbutamide) is also a sulfonylurea but is older and glibenclamide is much more potent and used commonly. Actrapid (soluble insulin) is fast acting but not a sulfonylurea.

130.

A patient has been given atropine sulfate for rapid relief of spastic colon symptoms. The use of this drug is contraindicated during the following disease:

Explanation

Glaucoma: increased intraocular eye pressure. Atropine sulfate (M-cholinoblocker) blocks the acetylcholine response of the ciliary muscle of the eye and of the circular smooth muscles of the iris, producing cycloplegia and mydriasis; it may increase intraocular pressure especially in patients with narrow-angle glaucoma. At low doses, it decreases cardiac rate (bradycardia). Arterial blood pressure is unaffected by atropine but at toxic levels, it will dilate the cutaneous vasculature. It is not effective in promoting healing of peptic ulcer. It can also relieve bronchial asthma. It is contraindicated because its going to further increase intraocular pressure.

131.

As an example of specific human parasites one can name Plasmodium falciparum, human pinworm and some others. The source of parasite invasion is always a human. Such specific human parasites cause the diseases that are called:

Explanation

Anthroponoses: diseases transmissible from human to human. Zoonoses: diseases transmissible from living animals to humans; formerly called anthropozoonoses. And diseases transmissible from humans to animals are called zooanthroponoses.

132.

In an experiment a dog had been conditioned to salivate at the sight of food and a flash of light. After conditioning the reflex, the light was then paired with the bell. The dog didn’t start to salivate. What type of inhibition was observed?

Explanation

The dog is conditioned to salivate at the sight of food and flash of light – that is a conditioned reflex that requires learning, memory and recall of previous experience. The established conditioned reflexes can be inhibited: externally and internally. External inhibition: disturbing factors like a stranger, sudden noise (the bell) or a strong smell can abolish the conditioned reflex and inhibit salivary secretion. This extra stimulus evokes the animal’s curiosity  and distracts the attention. Internal inhibition can be:

*Extinction of conditioned reflex: failure to reinforce the conditioned reflex by unconditioned stimulus.

 

*Differential: alteration of conditioned stimulus.

133. A newborn baby has numerous hemorrhages. Blood coagulation tests reveal increased prothrombin time. The child is most likely to have a disorder of the following biochemical process:

Explanation

image

Prothrombin time (PT) evaluates the extrinsic coagulation system down to the formation of the fibrin clot. Factors that are evaluated include factor VII, X, V, II, and I. PT is increased when a factor level is 30% - 40% of normal. Vitamin K-dependent factors include factor II, VII, IX and X; protein C and S. vitamin K is activated in the liver by epoxide reductase. Activated vitamin K ɣ-carboxylates each of the vitamin K-dependent factors. Carboxylated factors are now able to bind calcium, which are essential for interaction between the coagulation factors and platelet membranes.

Warfarin inhibits the enzyme vitamin K epoxide reductase. Neonates lack enteric bacteria which produce vitamin K.

134.

A 63-year-old male patient with bladder atony had been prescribed a medication, which he had been arbitrarily taking at a higher dose. The patient developed hyperhydration, salivation, diarrhea, muscle spasms. The prescribed drug relates to the following group:

Explanation

Acetylcholine is a neurotransmitter. It increases salivary secretion and stimulates intestinal secretions and motility. It increases bronchial secretions, increases tone of detrusor urinae muscle, causing expulsion of urine, stimulates ciliary muscle contraction (accommodation). It increases exocrine gland secretions (e.g. lacrimal, salivary, gastric acid); gut peristalsis, bladder contraction, bronchoconstriction, papillary sphincter muscle contraction (miosis). Poisoning by cholinomimetics causes diarrhea, urination, miosis, bronchospasm, bradycardia, excitation of skeletal muscle and central nervous system, lacrimation, sweating and salivation. Cholinomimetics are cholinergic agonists which produce analogous effects of acetylcholine.

135.

A 67-year-old male patient consumes eggs, pork fat, butter, milk and meat. Blood test results: cholesterol - 12,3 mmol/l, total lipids - 8,2 g/l, increased low-density lipoprotein fraction (LDL). What type of hyperlipoproteinemia is observed in the patient?

Explanation

Type IIa (familial hyperlipoproteinemia: ↑LDL and cholesterol. Autosomal dominant; due to absent or defective LDL receptors.

Type I:  ↑chylomicrons, triacylglycerol (TAG), cholesterol. Autosomal recessive; due to lipoprotein lipase deficiency or altered apolipoprotein C-II.

Type IV: ↑very low density lipoprotein (VLDL) and TAG. Autosomal dominant; due to hepatic overproduction of VLDL.

 

Type IIb: similar to Type IIa, except that VLDL is also increased and VLDL is normal for IIa.

136.

Tissue sampling of a 37-year-old male patient with chronic renal disease has revealed the following: sclerosis, lymphocytic and plasmocytic infiltration of renal pelvis and calices walls, dystrophy and atrophy of tubules. Remaining tubules are enlarged and stretched with colloid masses, epithelium is flattened out (\"scutiform\" or  \"shield-shaped\" kidney). What is the most likely diagnosis?

Explanation

Chronic pyelonephritis: microscopic changes involve predominantly tubules and interstitium. Tubules show atrophy and hypertrophy diffusely or dilatation. Dilated tubules may be filled with colloid crystals producing thyroidisation of tubules (thyroid-like). There is chronic interstitial inflammatory reaction, chiefly composed of lymphocytes, plasma cells, and macrophages with pronounced interstitial fibrosis. Renal pelvis and calyces are dilated and show marked chronic inflammation and fibrosis.

137.

During the histological study of cortical shaft, basophilic cells with developed synthesis organelles can be seen on the bone surface under the layer of fibers. These cells take part in bone tissue regeneration. What shaft layer are they located in?

Explanation

Osteoprogenitor cell is a resting cell that can transform into an osteoblast (bone forming cells) and secrete bone matrix. They are found on the external and internal surfaces of bones. They comprise the periosteal cells that form the innermost layer of the periosteum and the endosteal cells that line the marrow cavities. Electron micrographs reveal profiles of rough endoplasmic reticulum (rER) and free ribosomes as well as a small golgi apparatus and other organelles (developed synthesis organelles- rER, ribosomes, golgi apparatus).

138.

Autopsy of a 50-year-old male who had tuberculosis revealed a dense gray-white nidus in form of a nodule 2 cm in diameter in the subpleural portion of the upper right lobe. The pleura in this region was thickened, in the pleural cavity there was a small amount of serous hemorrhagic fluid. Histological study of the region revealed some glandular structures with signs of cellular atypia and abnormal mitoses, which were found within the fibrous connective tissue. What other pathology had developed in the lungs?

Explanation

Glandular structures – ‘adeno’.  Adenocarcinoma is an invasive malignant epithelial tumor with glandular differentiation or mucin production by the tumor cells. The lesions are usually peripherally located. Tumors (≤3cm) with a small invasive component (≤5mm) associated with scarring and a peripheral lepidic growth pattern are called microinvasive adenocarcinoma. Adenocarcinoma is one of the most common malignancies associated with tuberculosis.

139.

In course of an experiment there has been an increase in the nerve conduction velocity. This may be caused by an increase in the concentration of the following ions that are present in the solution around the cell:

Explanation

Nerve signals are transmitted by action potential (AP), which are rapid changes in the membrane potential that spread rapidly along the nerve fiber membrane. During the depolarization stage of an AP, the membrane suddenly becomes permeable to sodium ion (Na+), allowing tremendous number of positively charged Na+ to diffuse to the interior of the axon. A major function of voltage-gated calcium ion (Ca2+) channels is to contribute to the  depolarizing phase on the action potential in some cells. Although the gating of calcium channel is slow (slow channels), in contrast to the fast sodium channels. Therefore, sodium channels play a key role in initiation and conduction of action potentials.

140. Hepatitis B is diagnosed through laboratory tests that determine the presence of HBA-DNA in blood serum of the patient. What reference method is applied for this purpose?

Explanation

Polymerase chain reaction: molecular biology laboratory procedure used to amplify a desired fragment of DNA. Useful as a diagnostic tool. ELISA detect presence of either a specific antigen or a specific antibody in a patient’s blood sample.

141.

On the 24th day since the onset of disease, a male patient diagnosed with typhoid fever and undergoing treatment in an infectious diseases hospital has suddenly developed clinical presentations of acute abdomen leading to the death of the patient. During autopsy peritonitis has been revealed, with numerous ulcers covering the colon mucosa and reaching as deep as muscular and, in places, serous tunic. The ulcers have smooth edges and even floor. The intestinal wall is perforated. What stage of typhoid fever has the lethal complication arisen at?

Explanation

The changes in typhoid fever can be local and generalized. Local changes: has 5 stages. Each stage takes approximately one week.

* medullar swelling                                                                                   

* necrosis (7-10 days)

* ulcer formation (unclear, dirty ulcers)

* clean ulcer

* healing (recovery)

 

Clean ulcers has regular shape without necrotic tissue. In this stage the perforation can develop.

142.

An HIV-positive patient’s cause of death is acute pulmonary insufficiency resulting from pneumonia. Pathohistological investigation of lungs has revealed interstitial pneumonia, alveolocyte desquamation and their metamorphoses: alveolocyte enlargement, large intranuclear inclusions surrounded by lightly-colored areas. Transformed cells resemble owl’s eye. Name the pneumonia causative agent:

Explanation

Cytomegalovirus (CMV) cause congenital infection, mononucleosis, pneumonia, retinitis. Infected cells have characteristic “owl-eye” inclusions. Transmitted congenitally and by transfusion, sexual contact, saliva, urine, transplant. Interstitial pneumonia is mainly caused by viruses or mycoplasma. CMV is responsible for the most common viral opportunistic infection in persons with HIV infection.

143. The organisms to be identified have a nucleus surrounded by a nuclear membrane. Genetic material is concentrated predominantly in the chromosomes which consist of DNA strands and protein molecules. These cells divide mitotically. Identify these organisms:

Explanation

The fundamental difference between eukaryotes and prokaryotes is that eukaryotes do have ‘true’ nuclei containing their DNA, whereas the genetic material in prokaryotes is not membrane-bound. Eukaryotes contain multiple linear chromosomes with large amount of noncoding and repetitive DNA.

144. A 37-year-old male patient developed pseudoarthrosis after a closed fracture of the femur. Specify the type of regeneration in the patient:

Explanation

145.

Nucleolar organizers of the 13-15, 21, 22 human chromosomes contain about 200 cluster genes that synthesize RNA. These regions of chromosomes bear the information on the following type of RNA:

Explanation

Nucleolar organizer is a chromosomal region around which the nucleolus forms. In humans, nucleolar organizer contain genes for rRNA (ribosomal RNA) clustered on the short arms of chromosomes 13, 14, 15, 21, and 22.

146.

The resuscitation unit has admitted a patient in grave condition. It is known that he had mistakenly taken sodium fluoride which blocks cytochrome oxidase. What type of hypoxia developed in the patient?

Explanation

Tissue or histotoxic hypoxia is the type of hypoxia produced by the inability of tissues to utilize O2 which results from tissue poisoning. These poisonous substances destroy the cellular oxidative enzymes and there is a complete paralysis of cytochrome oxidase system.

 

Hemic hypoxia: connected with hemoglobin (Hb) quantity or inhibition of its functions. Cardiovascular or circulatory hypoxia arises during the disturbance of blood circulation due to heart and vessels pathology. Hypoxic hypoxia develops during the decrease of barometric pressure that is accompanied with decrease of pO2. Respiratory hypoxia arises as a result of respiratory insufficiency due to alveolar hypoventilation, disturbances of lungs blood supply, disturbances of gases diffusion in lungs.

147. Steatosis is caused by the accumulation of triacylglycerols in hepatocytes. One of the mechanisms of this disease development is a decrease in the utilization of VLDL neutral fat. What lipotropics prevent the development of steatosis?

Explanation

IMG_9833

Hepatic steatosis can occur when humans are deprived of choline.

Choline + Phosphatidic acid → Phosphatidylcholine (lecithin, PC). In the liver PC can also be synthesized from phosphatidylserine (PS) and phosphatidylethanolamine (PE), when free choline levels are low, because it exports significant amounts of PC in bile and as a component of serum lipoproteins (needed for fat metabolism)

PS → PE →→→ PC. 3 methylation reactions between PE and PC. S-adenosylmethionine is the methyl group donor. If choline, phosphatidylcholine or methionine is deficient, there will be abnormal phospholipid synthesis, oxidative damage caused by mitochondrial dysfunction, lipoprotein secretion (remember, if VLDL cannot be secreted it will be accumulated & cause fatty liver degeneration as seen in hepatic steatosis). PC is also a major lipid component of lung surfactant.

Vitamin B12 is a cofactor for homocysteine methyltransferase and methylmalonyl-CoA mutase. Both vitamin B12 and B6 are needed in fatty acid (FA) metabolism.

148. A 2-year-old boy is diagnosed with Down syndrome. What chromosomal changes may be the cause of this disease?

Explanation

krushkrok No149 (2014)FullSizeRender (1)

Chromosomal disorders: trisomy 21 (down’s syndrome); trisomy 18 (edward’s syndrome); trisomy 13 (patau’s syndrome); monosomy X (turner’s syndrome-XO); trisomy X (XXX);

normal female (XX); normal male (XY)

149.

A patient who has recently arrived from an endemic area presents with elevated body temperature, headache, chills, malaise, that is with the symptoms which are typical for a common cold. What laboratory tests are necessary to prove or to disprove the diagnosis of malaria?

Explanation

For malaria diagnosis, microscopy of blood smears, utilizing blood films; it is the most economic, preferred and reliable diagnosis of malaria because each of the 4 major parasite species has distinguishing characteristics (Plasmodium vivax, ovale, falciparum and malariae).

150.

A patient has severe blood loss caused by an injury. What kind of dehydration will be observed in this particular case?

Explanation

Dehydration (hypohydration, hypohydria, exicosis): in dehydration, the extracellular fluid (ECF) and sodium ions are lost.

*Isoosmolar dehydration: based on proportional volume decrease of fluids and electrolytes e.g. acute renal failure (stage of polyuria), blood loss, burns etc.

*Hypoosmolar dehydration (salt deficit) develops due to diarrhea, vomiting, sweating, adrenal insufficiency etc.

 

*Hyperosmolar dehydration (water deficit) develops due to the loss of the fluid which lacks electrolytes e.g. in diabetes insipidus, hyperventilation

151.

A 30-year-old patient has dyspnea fits, mostly at night. He has been diagnosed with bronchial asthma. What type of allergic reaction according to the Gell-Coombs classification is most likely in this case?

Explanation

FullSizeRender (17)

Type I (Immediate, Anaphylaxis, Reagin): IgE (immunoglobulin E)-dependent activation of mast cells/basophils, usually accompanied by eosinophilia e.g. urticaria (hives), hay fever, asthma (wheezing), rhinitis and conjunctivitis (stuffy nose and itchy eyes; usually seasonal)

Type II (cytotoxic): antibody dependent reactions e.g. Goodpasture syndrome, Myasthenia gravis, Graves disease, ABO hemolytic disease of newborn etc.

Type III (immune-complex): deposition of antigen-antibody complexes e.g. systemic lupus erythromatous (SLE), Arthus reaction, serum sickness, poststreptococcal glomerulonephritis etc.

Type IV (cell mediated, delayed): antibody-independent T-cell mediated reactions e.g. positive mantoux reaction (tuberculin test), hashimoto’s thyroiditis or transplant rejection etc.

152. Decarboxylation of glutamate induces production of gamma-aminobutyric acid (GABA ) neurotransmitter. After breakdown, GABA is converted into a metabolite of the citric acid cycle, that is:

Explanation

\"image\"

GAD- Glutamate decarboxylase

GABA-T – GABA transaminase

SSADH- succinate semialdehyde dehydrogenase

     Certain amino acids undergo decarboxylation that means the removal of their α-carboxyl group resulting in liberation of CO2 and formation of biogenic amines. Biogenic amines are physiologically active substances such as hormones, neurotransmitters etc. decarboxylation of amino acids:

·        Tryptophan → Niacin → NAD+/NADP+

Tryptophan → Serotonin →Melatonin

·        Histidine → Histamine

·        Glutamine → GABA

Glutamine → Glutathione

 
153.

An outbreak of an intestinal infection occurred in a kindergarten on the eve of New Year holidays. Bacteriological examination of patients’ faeces didn’t reveal any pathogenic bacteria. Electron microscopy revealed roundish structures with clear outer edges and a thick core resembling a wheel. Specify the most likely causative agent of this infection:

Explanation

image

Rotavirus is the most important global cause of infantile gastroenteritis, is a segmented dsRNA virus (a reovirus). Major cause of acute diarrhea during winter, especially in day care centers, kindergartens. ‘Roundish structure like a wheel’

Adenovirus causes febrile pharyngitis (sore throat), acute hemorrhagic cystitis, pneumonia, conjunctivitis (pink eye).

Coxsackievirus (hand-foot-mouth disease): oval-shaped vesicles on palms and soles; vesicles and ulcers in oral mucosa.

E. coli and Proteus vulgaris are bacteria; from the question, bacteriological examination of patient’s faeces didn’t reveal any pathogenic bacteria.

154. A smear of streptobacillus preparation stained by Ozheshko method has been studied microscopically with oil immersion. What structural feature of the bacteria has been studied?

Explanation

Ozheshko – Spores; Burri gins – Capsule
155.

After a road accident a victim has tachycardia, arterial blood pressure 130/90 mm Hg, tachypnoe, the skin is pale and dry, excitation of central nervous system is observed. What shock stage is the patient most likely in?

Explanation

Pirogov’s stages of shock

* Erectile phase: is characterized by strong motor agitation, sweating, tremor of skeletal muscles, staggering gait, frequent urination, transient increase in blood pressure, heart rate and breath rate increases, body temperature also. Painful impulses reach CNS.

 

* Torpid phase: decompensation in CNS leads to deep oppression. Patient is motionless, does not answer questions or answers very silently and with long time of delay, reflexes are lowered or absent.

156. Autopsy has revealed shrunken kidneys weighing 50 mg, with fine-grained surface and uniformly thinned substance. Microscopic investigation has shown the thickening of arteriole walls due to accumulation of homogeneous anhistic pink-coloured masses in them. Glomerules were undersized, sclerotic, with atrophied tubules. What disease are these changes characteristic of?

Explanation

Primary shrunken kidney is a renal form of essential hypertension. It is characterized by chronic arteriolosclerotic nephrosclerosis. Macroscopically, both kidneys are affected equally and are reduced in size and weight. The surface of the kidney is finely granular. The cut surface shows firm kidney and narrowed cortex. Microscopically, there is variable degree of atrophy of parenchyma; these include glomerular shrinkage, deposition of collagen in Bowman’s space, periglomerular fibrosis.

Secondary shrinkage of kidneys is found in chronic glomerulonephritis.         
157.

A fixed-run taxi passenger has a sudden and expressed attack of tachycardia. A doctor travelling by the same taxi has managed to slow down his heart rate by pressing upon the eyeballs and thus causing the following reflex:

Explanation

Aschner’s reflex (press on eyeball) → ↓heart rate. This is mediated by nerve connections between the ophthalmic branch of trigeminal cranial nerve via the ciliary ganglion and the vagus nerve of parasympathetic nervous system.

 

Goltz reflex (press or blow to the epigastric region) → ↓heart rate.

158.

Histological examination of biopsy samples taken from the thickened edges of a gastric ulcer revealed small clusters of small, markedly atypical hyperchromatic epithelial cells that were localized in the overdeveloped stroma. Specify the tumor: 

Explanation

Undifferentiated carcinoma: tumor is composed exclusively of undifferentiated cells with deeply stained pleomorphic nuclei and scanty cytoplasm, with no evidence of differentiation toward squamous carcinoma or adenocarcinoma. Pure undifferentiated carcinoma of the gastric cardia is rarely seen.

 

Scirrhous carcinoma: in this pattern, the stomach wall is thickened due to extensive desmoplasia. Therefore, we have scirrhous undifferentiated carcinoma.

159. A 10-year-old child was found to have a congenital hypoplasia of the left kidney. Ultrasound examination revealed that the right kidney was markedly enlarged and had regular shape. No functional disorders were revealed. Specify the process that developed in the right kidney:

Explanation

Pathological hypertrophy (hyperplasia) types: neurohumoral; working; compensatory reparative; vicarious (substitutional); hypertrophic vegetations

* Vicarious (substitutional) hypertrophy: following nephrectomy (or congenital hypoplasia) on one side in a young patient, there is compensatory hypertrophy as well as hyperplasia of the nephrons of the other kidney.

* Working hypertrophy: cardiac muscle develop hypertrophy at some cardiac diseases such as systemic hypertension, aortic stenosis or insufficiency; skeletal muscles (e.g. biceps) in athletes. ↑ workload → hypertrophy
160.

In allergic diseases, a dramatic increase in basophilic leukocyte number in patients’ blood is observed. This phenomenon is due to the following basophil function:

Explanation

Remember that the first contact of allergen with immune cells occurs in the immunological stage resulting in IgE bound to the surface of mast cells. Pathochemical stage of anaphylactic (Type I- allergic) reaction: now, the next exposure to the same antigen results in fixing of the antigen to mast cell-bound IgE. This process leads to mast cell degranulation with the discharge of preformed or primary mediators (histamine, heparin, serotonin etc) and synthesis of secondary mediators (prostaglandins, leukotrienes and cytokines).

161.

A 47-year-old male patient developed intestinal colic against the background of essential hypertension. In this situation it would be most efficient to arrest the colic by using drugs of the following group:

Explanation

Only myotropic antispasmodics (muscle relaxants) can effectively manage both intestinal colic and essential hypertension. Because sympathomimetics and adrenomimetics will exacerbate hypertension the more. Anticholinesterase agents and M-cholinomimetics will ↑acetylcholine effects in neuromuscular junction → ↑arteriolar smooth muscle tone → exacerbate hypertension. But the myotropic antispasmodics can ↓ muscle tone in intestinal smooth muscle (thereby relieving intestinal colic) and also ↓ arteriolar smooth muscle tone (thereby relieving essential hypertension).

162.

A patient has been admitted to the contagious isolation ward with signs of jaundice caused by hepatitis virus. Which of the symptoms given below is strictly specific for hepatocellular jaundice?

Explanation

In viral hepatitis, there is generalized liver dysfunction involving uptake and conjugation of unconjugated bilirubin, secretion of conjugated bilirubin into bile ducts, and recycling of urobilinogen. Alanine transaminase (ALT) and Aspartate transaminase (AST) are increased (↑), but ALT is higher than AST and there is a slight ↑ in alkaline phosphatase (ALP) and ɣ-glutamyltransferase (GGT).

ALT is a specific enzyme for liver necrosis; present in the cytosol. ALT>AST: viral hepatitis.

 

AST is present in the mitochondria. Alcohol damages mitochondria AST>ALT indicates alcoholic hepatitis

163. An underage patient has signs of achondroplasia (dwarfism). It is known that this is a monogenic disease and the gene that is responsible for the development of such abnormalities is a dominant one. The development of that child’s brother is normal. Specify the genotype of the healthy child:

Explanation

A – gene for achondroplasia dwarfism

        a – normal gene

AA (homozygous) and Aa (heterozygous) will express the disease because the gene controlling the disease is dominant.

 

Only a recessive homozygous gene (aa) can express the normal phenotype

164. A patient has acute bronchitis. The fever up to 38, 5oC had lasted for a week, presently there is a decrease in temperature down to 37, 0oC. Specify the leading mechanism in the 3rd stage of fever:

Explanation

image

Stages of fever:

I. Stadium Increment (↑of temperature)

II. Stadium Fastigii (stable, but high temperature; saving of the increased temperature)

III. Stadium Decrementi (decline of temperature)

Temperature decreasing mechanisms in the 3rd stage: vasodilation of skin blood vessels (peripheral vasodilation); decrease of heat production; sweating

165. A 35-year-old female patient has undergone biopsy of the breast nodules. Histological examination has revealed enhanced proliferation of the small duct and acini epithelial cells, accompanied by the formation of glandular structures of various shapes and sizes, which were located in the fibrous stroma. What is the most likely diagnosis?

Explanation

FullSizeRender (21)

Adenoma: benign epithelial tumor from the epithelium of the glands and glandular organs. More often they can be found in the breast, thyroid gland, liver, ovaries, prostate gland, GIT. In the question, it is located in the fibrous stroma – FIBROADENOMA.

166. At a bacteriological laboratory animal skins are analyzed by means of Ascoli precipitation test. What is detected if the reaction is positive?

Explanation

Ascoli’s test or reaction is for detection of Anthrax antigens. Positive Ascoli’s test confirms the diagnosis, using an anthrax antiserum.

167.

Examination of the duodenal contents revealed some pear-shaped protozoa with two nuclei and four pairs of flagella. The organisms had also two axostyles between the nuclei and a ventral adhesive disc. What protozoan representative was found in the patient?

Explanation

imageimage

Giardia lamblia is pear shape or “clownface” with two nuclei, 4 pairs of flagella. The two nuclei are outlined by adhesive discs. Transmitted by cysts in water. The cysts are oval, have 4 nuclei and have clearly visible axostyles. It causes Giardiasis – bloating, flatulence, foul-smelling, fatty diarrhea. Diagnosis: trophozoites or cysts in stool.

168. A 36-year-old female patient has a history of B2-hypovitaminosis. The most likely cause of specific symptoms (epithelial, mucosal, cutaneous, corneal lesions) is the deficiency of:

Explanation

Vitamin B2 (riboflavin): component of flavins FAD and FMN – which are the coenzyme forms. Deficiency causes cheilosis (inflammation of lips, scaling and fissures at the corners of the mouth), corneal vascularization.
169. A 54-year-old female was brought to the casualty department after a car accident. A traumatologist diagnosed her with multiple fractures of the lower extremities. What kind of embolism is most likely to develop in this case?

Explanation

Embolism is occlusion of a vessel by material travelling in the circulation.

* Fat/Adipose embolism: obstruction of arterioles and capillaries by fat globules constitutes fat embolism. It may occur following severe fracture trauma to bones, inflammation of bones and soft tissues, fatty liver

* Thromboembolism: a detached thrombus or part of thrombus which may arise in the arterial or venous circulation.

* Gas embolism: two main forms of gas embolism are air embolism and decompression sickness. Air embolism is usually due to accidental pumping of air into the venous circulation during intravenous (IV) injection or transfusion ( bubble – air escaped).

 

Tissue embolism: fragments of tissue.

170.

A blood drop has been put into a test tube with 0,3% solution of NaCl. What will happen to erythrocytes?

Explanation

image

The intracellular fluid of erythrocytes is a solution of salts, glucose, protein and hemoglobin. A 0.9% NaCl solution is said to be isotonic: when blood cells reside in such a medium, the intracellular fluid (ICF) and extracellular (ECF) are in osmotic equilibrium across the cell membrane and there is no net influx or efflux of water.

On the other hand, in a hypnotic environment (e.g. 0.3% NaCl or distilled water), an influx of water occurs; the cells swell, the integrity of their membranes is disrupted, allowing the escape of their hemoglobin (hemolysis) which dissolves in the external medium.

When subjected to hypertonic media (e.g 1.3% NaCl) the cells lose their normal biconcave shape, undergoing collapse (shrinkage leading to crenation) due to the rapid osmotic efflux of water.

171.

There are cortical and medullary substances separated by connective tissue layer in the endocrine gland specimen. Parenchyma cells make up three zones in cortical substance, with rounded masses in the superficial zone, parallel chords in the middle one, reticular structure of cell chords in the deep one. What gland is it?

Explanation

FullSizeRender (22)

The adrenal gland (suprarenal gland) has a secretory parenchymal tissue organized into cortical and medullary regions. Adrenal cortex (derived from mesoderm; steroid-secreting portion) and medulla (derived from neural crest; catecholamine-secreting portion). The adrenal cortex is divided into 3 zones on the basis of arrangement of its cells:

* Zona glomerulosa (15%): arranged in closely packed ovoid clusters; secretes aldosterone.

* Zona fasciculata (80%): large and polyhedral; arranged in long straight cords; secretes cortisol.

* Zona reticularis (5-7%): cells are arranged in anastomosing cords separated by fenestrated capillaries; secretes androgens.

172. Histological specimen of a 10-day human embryo represents 2 contacting sacs (amniotic and yolk sacs). Specify the structure that separates the amniotic cavity from the yolk sac:

Explanation

krushkrok No172 (2014)

On the 8th day of development embryoblast differentiate into two layers:

* a layer of small cuboidal cells adjacent to the blastocyst cavity – hypoblast layer

* a layer of high columnar cells adjacent to the amniotic cavity – epiblast layer.

Together the layers form a flat disc – the embryonic shield which separates the amniotic cavity from the yolk sac.

173.

An electron micrograph shows a cell of neural origin. The terminal portion of the cell dendrite has cylindrical shape and consists of 1000 closed membrane disks. What cell is represented by the micrograph?

Explanation

Retina contains the visual receptors – rods and cones are composed of 4 structures: outer segment, inner segment, cell body and synaptic terminal. The outer segment of rod cell (which is the terminal portion as described in the question) is formed by the modified cilia and it contains a pile of freely floating flat membranous disks. There are about 1000 disks in each rod. Disks in rod cells are closed structures and contain the photosensitive pigment, the rhodopsin. The outer segment of cones is small and conical. It does not contain separate membranous disks as in rods. In cone, the infoldings of cell membrane form saccules, which are the counterparts of rod disks.

174.

Disruption of nerve fiber myelinogenesis causes neurological disorders and mental retardation. These symptoms are typical for hereditary and acquired alterations in the metabolism of:

Explanation

Sphingolipids are commonly believed to protect the cell surface against harmful environmental factors by forming a mechanically stable and chemically resistant outer leaflet of the plasma membrane lipid bilayer. Certain complex glycosphingolipids (cerebrosides) are found predominantly in the brain and peripheral nervous tissue, with high concentrations in the myelin sheath. Gangliosides (most complex glycosphingolipids) are found primarily in the ganglion cells of the CNS, particularly at the nerve endings. Sphingolipidoses or disorders of sphingolipid metabolism, have particular impact on neural tissue e.g. Niemann-Pick disease, Fabry disease, Krabbe disease, Gaucher disease, Tay sachs disease. All of the diseases are autosomal recessive except Fabry disease which is X-linked and all can be fatal in early life.

175.

A patient underwent surgical removal of a cavitary liver lesion 2 cm in diameter. It was revealed that the cavity wall was formed by dense fibrous connective tissue; the cavity contained muddy, thick, yellowish-greenish fluid with an unpleasant odor. Microscopically, the fluid consisted mainly of polymorphonuclear leukocytes. What pathological process are these morphological changes typical for?

Explanation

Chronic abscess has internal pyogenic membrane; middle – granulation tissue; external – fibrous tissue membrane. Acute abscess basically has only a pyogenic membrane no fibrous tissue membrane. Phlegmon: diffuse purulent inflammation which occurs along muscular fibers, tendons, fascias etc. Empyema is a purulent inflammation of serous membranes (e.g. empyema of pleura). Both phlegmon and empyema does not involve the formation of cavity (“cavitary lesion”).

176.

A patient complains of photoreception disorder and frequent acute viral diseases. He has been prescribed a vitamin that affects photoreception processes by producing rhodopsin, the photosensitive pigment. What vitamin is it?

Explanation

Rhodopsin or visual purple is the photosensitive pigment of rod cells. It is made up of a protein called opsin and a chromophore. Opsin present in rhodopsin is known as scotopsin. Chromophore is a chemical substance that develops colour in the cell. Chromophore present in the rod cells is called retinal. Retinal is the aldehyde of vitamin A or retinol. Vitamin A is the name given to a group of related compounds that include retinol (vitamin A alcohol); retinal (vitamin A aldehyde) and retinoic acid (vitamin A acid). Rod cells are responsible for dim light vision or night vision or scotopic vision.

177.

A 7-year-old boy got ill with diphtheria. On the third day he died of asphyxiation. At autopsy the mucosa of the larynx, trachea and bronchi had thickened, edematous, lustreless appearance and was covered with gray films which could be easily removed. Specify the type of laryngeal inflammation:

Explanation

There are two types of fibrinous inflammation: croupous and diphtheric fibrinous inflammation. Usually croupous inflammation develops on the columnar epithelium. In this case the fibrinous membranes unfix easily, without any effort. Diphtheric fibrinous inflammation develops on the squamous or intermediate epithelium and the fibrinous membranes unfix with difficulties and may even bleed when trying to unfix it.

178. A child has a history of hepatomegaly, hypoglycemia, seizures, especially on an empty stomach and in stressful situations. The child is diagnosed with Gierke’s disease. This disease is caused by the genetic defect of the following enzyme:

Explanation

Glycogen storage diseases: 12 types, all resulting in abnormal glycogen metabolism and an accumulation of glycogen within cells. Type I (Von Gierke disease): findings – severe fasting hypoglycemia, ↑↑glycogen in liver, ↑blood lactate, ↑triglycerides, ↑uric acid and hepatomegaly. Deficient enzyme is glucose-6-phosphatase. It is autosomal recessive.

Deficiency of amyloid-1,6-glycosidase (Type III-Cori’s disease); deficiency of glycogen phosphorylase (Type V-McArdle’s; Type VI-Hers’)
179.

A public utility specialist went down into a sewer well without protection and after a while lost consciousness. Ambulance doctors diagnosed him with hydrogen sulfide intoxication. What type of hypoxia developed?

Explanation

Hemic hypoxia (blood hypoxia): may be connected with hemoglobin (Hb) quantity or inhibition of its functions. It is observed during anemia of different genesis; also during poisoning with carbon monoxide, nitrates, sulfa drugs (H2S) and other substances. In this case we will observe inactivation type of hemic hypoxia with the acquired hemoglobinopathies development. Acquired hemoglobinopathies are characterized by formation of carboxyhemoglobin and methemoglobin.

180.

A child with a normal karyotype is diagnosed with cleft lip and hard palate, defects of the cardiovascular system, microcephaly. The child’s mother suffered rubella during pregnancy. This pathology in the child may be an example of:

Explanation

Genocopy is a trait that is a phenotypic copy of a genetic trait but is caused by a different genotype. A phenotype that appears identical to another phenotype but that is caused by a different genetic mechanism e.g. the cleft lip and palate is identical to patau’s syndrome (trisomy 13) but in this case the patient has a normal karyotype. Phenocopy is a phenotype that is not genetically determined but mimics one that is. It is the opposite of genocopy.

181.

A 28-year-old patient undergoing treatment in the pulmonological department has been diagnosed with pulmonary emphysema caused by splitting of alveolar septum by tissular tripsin. The disease is caused by the congenital deficiency of the following protein:

Explanation

Emphysema: permanent enlargement of all or part of the respiratory unit. Causes include smoking cigarette (most common cause) and α1-antitrypsin (AAT) deficiency. α1-antitrypsin is a protease inhibitor; also referred to as α1-proteinase inhibitor because it inhibits a wide variety of proteases (including trypsin as the name implies).

182.

A   patient   with   signs   of osteoporosis and urolithiasis has been admitted to the endocrinology department. Blood test has revealed hypercalcemia and hypophosphatemia. These changes are associated with abnormal synthesis of the following hormone:

Explanation

image

Parathyroid hormone: secreted by chief cells of parathyroid gland. Effects include:

↑bone resorption of Ca2+ and PO43- → ↑their plasma levels

↑kidney reabsorption of Ca2+ in distal convoluted tubule → ↑ Ca2+ plasma level

↓reabsorption of PO43- in proximal convoluted tubule → ↓ PO43- plasma levels

↑Calcitriol (vit D3) production by stimulating kidney 1α-hydroxylase in proximal convoluted tubule. It increases Ca2+ and PO43- absorption in the intestine.

In general, parathyroid hormone ↑ Ca2+ plasma level but ↓ PO43- plasma levels. Abnormal synthesis (↑synthesis) of parathyroid hormone can lead to hypercalcemia and hypophosphatemia.

Calcitonin is secreted by parafollicular cells (C cells of the thyroid gland). It ↓bone resorption of Ca2+. It opposes actions of parathyroid hormone. But its not important in normal Ca2+ homeostasis. Calcitriol ↑ circulating Ca2+ ions as a means of enhancing intestinal absorption of calcium (NB: Calcitriol production is dependent on parathyroid hormone). Aldosterone: reabsorption of Na+, excretion of K+. Cortisol – glucocorticoid: ↑ blood pressure and gluconeogenesis; ↓ inflammatory and immune responses; ↓ bone formation (↓osteoblast activity).

183.

During a surgery for femoral hernia a surgeon operates within the boundaries of femoral trigone. What structure makes up its upper margin?

Explanation

krushkrok No184 (2014)krushkrok No184a (2014)

Boundaries of femoral triangle:

*Superiorly: inguinal ligament

*Medially: medial border of adductor longus muscle

*Laterally: medial border of the Sartorius muscle

*Roof: fascia lata

*Floor: adductor longus muscle, pectineus muscle and the iliopsoas muscle

184.

A 19-year-old victim has been delivered to the casualty department with a cut wound of the trapezius muscle. Which of the cervical fasciae forms a sheath for this muscle?

Explanation

IMG_9912

Fascia of the neck:

Superficial cervical fascia: investing fascia; it lies beneath the platysma muscle and surrounds all of the deeper structures of the neck. Within the borders of the lateral neck triangle, the superficial layer of the cervical fascia splits; its superficial sheet runs towards the back, encloses the trapezius forming its fascial sheath; its deeper sheet attaches to the transverse processes of the cervical vertebrae.

Deep cervical fasciae: pretracheal; prevertebral; fascia of the carotid sheath

185.

A patient with acne has been prescribed doxycycline hydrochloride. What recommendations should be given to the patient, while he is taking this drug?

Explanation

Toxicity/adverse effect of tetracycline: GI distress, discoloration of teeth and inhibition of bone growth in children, fatal hepatotoxicity; photosensitivity (may cause severe sunburn when patients is exposed to sun or UV rays). Therefore avoid long stay in sun. It is contraindicated in pregnancy.

186. A 30-year-old patient with a past history of virus B hepatitis complains of prolonged nosebleeds. What drug will be most efficient in remedying this condition?

Explanation

Vikasol (Menadioni natrii bisulfis) is a synthetic water soluble analogue of Vitamin K. it’s a coagulant. It participates in the formation of prothrombin. It promotes normalization of blood clotting. Indications include acute hepatitis, parenchymal and capillary bleeding, obstructive jaundice.

 

Dipiridamol – antiplatelet; platelet aggregating inhibitor (it’ll further exacerbate the nosebleed); Folic acid (Vit B9) is used to treat megaloblastic, macrocytic anemia; Fraxiparin – anticoagulant; Asparcam (panagin): source of K+ and Mg2+, prescribed for hypokalemia and hypomagnesemia. K+ and Mg2+ are not important for blood coagulation.

187. A patient has arterial hypertension. What long-acting drug from the group of calcium channel blockers should be prescribed?

Explanation

Amlodipine is a calcium channel blocker (long acting); from the dihydropyridine family.

 

Atenolol is a β-blocker (cardioselective); Reserpine and Octadine are sympatholytics; Pyrroxanum is an adrenergic antagonist/α-adrenoblocker (non-selective α1 and α2).

188. A patient has been diagnosed with ARVI. Blood serum contains immunoglobulin M. What is the stage of infection in this case?

Explanation

ARVI – Acute respiratory viral infection. High IgM level usually indicate an acute or primary infection because, it is the first antibody to appear in response to initial exposure to an antigen. Secondary or reinfection show an increase in IgG. IgM antibodies appear early in the course of an infection, this makes it useful in the diagnosis of infectious diseases. Demonstrating IgM antibodies in a patient’s serum indicates recent infection or in a neonate’s serum, it indicates intrauterine infection.

189. In a dysentery patient undergoing treatment in the contagious isolation ward, a significant increase in packed cell volume has been observed (60%). What other value will be affected by this change?

Explanation

Packed cell volume (PCV) or hematocrit is the volume in percent of red blood cells (RBCs) in blood; 45% for men; 40% for women. So a PCV of 60% indicates a significant increase in RBCs in the blood which will directly increase the thickness of blood and cause vessel congestion as a result of increase in blood viscosity. Increased RBCs directly increase blood viscosity. Volume of blood will increase but volume is more closely related to the fluid component of blood.

190. A patient complains of palpitation after stress. The pulse is 104 bpm, P-Q=0,12 seconds, there are no changes of QRS complex. What type of arrhythmia does the patient have?

Explanation

Sinus tachycardia is the increase in discharge of impulses from the sinoatrial (SA) node, resulting in increase in heart rate (heart rate increase up to 100beats/min).ECG is normal, except for short R-R interval.

 

Sinus bradycardia is the reduction in discharge of impulses from SA node resulting in decrease in heart rate. Heart rate is less than 60beats/min. Extrasystole is the premature contraction of the heart before its normal contraction. ECG is altered. Sinus arrhythmia is characterized by irregular generation of impulses and may be due to variations in the tone of the vagus nerve. ECG is altered.

191. A 30-year-old patient has undergone keratoplasty in the transplantation center, cornea has been taken from a donor, who died in a road accident. What kind of transplantation was performed?

Explanation

Allotransplantation: from an individual (donor) to another

Autotransplantation: from one part of the body to another (e.g. skin on thigh to         face).

Xenotransplantation and Heterotransplantation: from one specie to another (e.g. pig to human)

 

Explantation: removal of a body tissue/organ

192. A 29-year-old male with a knife wound of neck presents with bleeding. During the initial debridement of the wound the surgeon revealed the injury of a vessel found along the lateral edge of the sternocleidomastoid muscle. Specify this vessel:

Explanation

IMG_9913

External jugular vein is a subcutaneous vein that arises by the union of the anterior (which is the anastomosis with the retromandibular vein) and posterior (formed of the occipital and posterior auricular veins). External jugular vein crosses the sternocleidomastoid muscle laterally and at its midpoint, then opens into the venous angle – the junction point of the subclavian and internal jugular veins.

193. A histologic specimen represents an organ with walls comprised of mucous, submucous, fibrous-cartilaginous and adventitial membranes. Epithelium is multirowed and ciliated, muscular layer of mucous membrane is absent, submucous membrane contains serous-mucous glands, hyaline cartilage forms open circles. What organ has the described morphological features?

Explanation

The wall of the trachea consists of 4 layers:

*Mucosa: composed of a ciliated, pseudostratified epithelium and an elastic, fiber-rich lamina propria.

*Submucosa: composed of a slightly denser connective tissue than the lamina propria.

*Cartilaginous: composed of C-shaped (open circles) hyaline cartilages.

*Adventitia: composed of connective tissue that binds the trachea to adjacent structures.

 

Submucosal glands are composed of mucus-secreting acini. Bronchi (both segmental and lobar) have muscularis mucosa which is absent in trachea. No cartilage plates in bronchioles (terminal bronchiole). Cartilages in the larynx are arytenoids, epiglottis, cricoids, thyroid

194. In cancer patients who have been continuously receiving methotrexate, the target cells of tumor with time become insensitive to this drug. In this case, gene amplification of the following enzyme is observed:

Explanation

IMG_9914

Methotrexate is structurally related to folic acid and acts as an antagonist of that vitamin by inhibiting dihydrofolate reductase (in humans), which is the enzyme that converts folic acid to its active coenzyme form, tetrahydrofolic acid. Trimethoprim and pyrimethamine inhibit the same enzyme but in bacteria and protozoa respectively. This decreases dTMP, needed for DNA synthesis.

195. Pancreas is known as a mixed gland. Endocrine functions include production of insulin by beta cells. This hormone affects the metabolism of carbohydrates. What is its effect upon the activity of glycogen phosphorylase (GP) and glycogen synthase (GS)?

Explanation

krushkrok No196 (2014)

Insulin stimulates glycogen synthase by dephosphorylation. Insulin is an anabolic hormone (build up). So, it stimulates glycogen synthase to favor the synthesis of glycogen (glycogenesis) and inhibits glycogen phosphorylase used in glycogenolysis (glycogen breakdown). Glucagon and Epinephrine stimulates glycogen phosphorylase to break down glycogen to glucose and make energy available for “fight and flight” reactions.

196.

A patient has the oxyhemoglobin dissociation curve shifted to the left. What blood changes induce this condition?

Explanation

When the curve shifts to the right, there is decrease affinity of hemoglobin for O2 (this facilitates unloading of O2 to tissues). An increase in all listed factors (including H+) causes a shift of the curve to the right. A decrease in the factors (including H+) causes a shift of the curve to the left. Fetal hemoglobin has higher affinity for O2 than adult hemoglobin, so its dissociation curve is shifted to the left.

Right shift (↑): acidosis(↑H+); CO2; exercise; 2,3-BPG; Altitude; Temperature. Hence left shift will be (↓): alkalosis(↓H+); hypocapnia(↓CO2); temperature decrease.
197.

Administration of doxycycline hydrochloride caused an imbalance of the symbiotic intestinal microflora. Specify the kind of imbalance caused by the antibiotic therapy:

Explanation

Administration of doxycycline hydrochloride (antibiotic) can cross react with the normal microflora in the intestine thereby causing dysbacteriosis (an imbalance in the normal microflora). All broad spectrum antibiotics can be associated with dysbacteriosis.

198.

A patient with signs of emotional lability that result in troubled sleep has been prescribed nitrazepam. Specify the sleep-inducing mechanism of this drug:

Explanation

Nitrazepam is a hypnotic (tranquilizer) drug from the benzodiazepine family. Other drugs in this family include diazepam, lorazepam, triazolam, oxazepam etc. they facilitate GABAA action by increasing frequency of Cl- channel opening. They decrease rapid eye movement (REM) sleep.

199.

A patient has been found to have a marked dilatation of saphenous veins in the region of anterior abdominal wall around the navel. This is symptomatic of pressure increase in the following vessel:

Explanation

Portal hypertension:

Pathogenesis: resistance to intrahepatic blood flow due to intrasinusoidal hypertension. Anastomoses between portal vein (vena porta hepatis) tributaries and the arterial system. Complications: ascites, periumbilical venous collaterals (caput medusa); esophageal varices; congestive splenomegaly; hemorrhoids

 

↑in blood pressure in portal vein → portal hypertension 

200.

A patient with suspected tumor of lung had been admitted to the oncological department. Examination revealed localised pathology in the inferior lobe of the left lung. How many bronchopulmonary segments does this lobe have?

Explanation

krushkrok No100 (2014)

The inferior lobe of left lung has 5 segments: superior, medial basal, anterior basal, lateral basal, posterior basal segments.